You are on page 1of 138

NCCT Phlebotomy Review PG1

Success Strategies
Taking Ownership
As a new professional in the medical field, many new things will be expected of you. Your first
step towards reaching your goals is to “own” your destiny. Give your studies everything you’ve
got and be confident in your skills when it comes time to take your certification exam.
Click through the topics presented on this screen and come back from time to time as you
prepare for success.

Time management
Your success in studying for the certification exam may be largely dependent on how
well you manage your time. All of us have many obligations and demands on our time and
unless you develop a study plan, your preparation time disappears as the examination date
moves closer. Planning allows you to focus on your effort towards preparation.
Use a calendar to create a study schedule between now and the examination date. It
should list each exam category and how much time you plan to allot for study in each one. As
you check off each content subcategory and category, you will have the sense of
accomplishment that comes with knowing you are doing everything you can to score well on
your certification exam.
It is often helpful to begin a thorough review of exam content by tackling the subjects
that are most difficult for you first. Then, once you have conquered those subjects, move into a
thorough review of the areas in which you feel more confident. The last portion of your study
schedule should include a recap of any continuing areas of doubt or weakness, referring back to
your textbooks as needed. A good schedule should help eliminate the need to cram for the
exam. Cramming is not an effective study (or learning) technique and is never recommended.

Where and when to study


Select a Specific Study Place: Designate a specific area as your study place. Find a
place that is comfortable and has good lightning, but avoid becoming too relaxed to concentrate.
Having a dedicated study area may help your mind to get into study mode. Find a sturdy desk or
table that allows you to spread out your study materials. Quality study time should have minimal
distractions, so you should select a place where phones, tv, family and friends will not desterbe
you. You may even want to consider hanging a “Do Not Disturb” sign.
Select a specific study time: Choose a specific time to study every day or at least several
times a week. Determine a realistic number of hours per week you will study. Write your study
times on a calendar. Ideally, you should study when you are well rested, so your brain is more
alert. Consider the times of day that are most productive for you. Example if you are a morning
person, try to study in the morning. Don’t study to the point of exhaustion since that is not a
productive way to retain information.
Celebrate your progress: After each study session, check the date off your calendar. Each day
you study movies you are one day closer to achieving your goals. SOmetimes it helps to
promise yourself a small reward after checking off a certain number of dates. Your reward can
NCCT Phlebotomy Review PG2
be anything that motivates you: a movie, a nap, a walk, or a special snack. Little incentives like
this can make a big difference

Planning Ahead
Getting Organized: Before beginning a study session, have all your study and reference
materials available as you work through the online module. Have pencils and paper available to
record notes in an organized manner. Develop a positive attitude If you believe you will
succeed, you are more likely to make it happen. A positive attitude can result in more effective
study which reflects more confidence going forward on exam day.
Watch what you eat: Your diet during your examination preparation time is important. Try
to eat light, well-balanced meals before studying. If time is short, eat a piece of fruit, drink some
juice, or munch on a light healthy snack.

Study Groups
Strength in Numbers: Many students find working in a group to be helpful. Study groups
make it possible to share information and tips. One person’s weak area may be another’s
strength. Study groups can help you gain new perspectives.
Set some guidelines: If you decide to form a study group, it is helpful to establish some
guidelines since it is easy for distractions or side-conversations to distract or side-conversations
to disrupt group processes. Reach an agreement about socialization. It is usually best to set
aside a brief period at the beginning of the study session to visit with one another. Once you can
then have productive study time together. If you have a close group, some additional time for
conversation at the end of a study session can be a reward for the group!

The day before your exam


Learn the testing site location: Make sure you know how to get to the examination
location. If you are unfamiliar with the location, you might consider driving there before the
actual exam day to learn the route. Listen to the weather and traffic reports to ensure that you
can arrive at least 30 minutes before the examination start time. Always allow yourself extra
travel time in case there are unexpected delays.
Get everything you need for the examination the night before the exam / photo id,
authorization to test, and/or anything else that is required. Place all of these items in one
location where you will be sure to take them with you when you need to leave for the
examination.
Eat a healthy diet and get plenty of rest: The evening before your exam, eat a well
balanced meal and get a good night’s sleep. The more rested you are, the more you will be able
to think clearly. Set your alarm so that you have plenty of time to get up, get ready, and get to
the test site at least 30 minutes before the start of the examination.

Exam Day
You are going to do great: Eat a good breakfast. Make sure you are hydrated. Wear
comfortable clothes and layer up so you can easily adapt to a variety of room temperatures.
Arrive early enough to relax a bit before the exam. Before your exam begins , take a deep
breath and remind yourself you have done everything you could to prepare. You are ready
NCCT Phlebotomy Review PG3
Time to take your exam: Listen carefully as the examination proctor reads directions.
Even if you find them a bit boring, they are very important. If you do not understand any of the
information given, ask for clarification before you start. When directed by the proctor, read the
examination directions carefully. Do not start the exam until the proctor tells you to do so.

Taking the Examination


Your Exam - A Mental Checklist!
Listen carefully as the examination proctor reads directions. ~ Even if they are a bit
boring, they are important. If you do not understand any of the information given, ask for
clarification before you start. When directed by the proctor, read the examination directions
carefully. Do not start the exam until the proctor tells you to do so.
Estimate how much time you have to answer each question. ~ For example, if the
maximum time allowed for the exam is 3 hours and there are 165 questions, you have
approximately 1 minute per question (180 minutes divided by 165 questions = 1.09 minutes per
question).
Read each test item or question carefully. ~ Try to identify the correct answer in your
mind before looking at the answer options. If you cannot find it, make sure you did not misread
the question. Look at it again.
If you cannot answer it outright, eliminate choices that are obviously incorrect. ~ Narrow
down your options. Consider all possibilities before choosing your final answer.
Some questions use negatives like “not, never, or nor” and others use qualifiers such as
“all, always, only, or except.” ~ Pay special attention to these questions, to avoid being
confused.
Do not spend too much time on any single question. ~ If you cannot identify any answer
for a question, note the question number so you can return to it later. Sometimes, you will
remember the information later…or another question on the test may help you remember a fact
that can lead you to the correct answer.
Keep track of your time and pace yourself ~ Do you need to speed up or slow down?
Any extra time you have can be used to review questions of which you are unsure of the
answer. Periodically assess how many questions you have left to answer.
Answer all questions on the examination. ~ There is no penalty for guessing. Make sure
to answer every question, even if it is a guess

2020 National Patient Safety Goals


Identify patients correctly
NPSG.01.01.01
Use at least two ways to identify patients. For example, use the patient's name and date of birth.
This is done to make sure that each patient gets the correct medicine and treatment.
Use medicines safely
NPSG.03.04.01
Before a procedure, label medicines that are not labeled. For example, medicines in syringes,
cups and basins. Do this in the area where medicines and supplies are set up.
NCCT Phlebotomy Review PG4
NPSG.03.05.01
Take extra care with patients who take medicines to thin their blood.
NPSG.03.06.01
Record and pass along correct information about a patient's medicines. Find out what medicines
the patient is taking. Compare those medicines to new medicines given to the patient. Make
sure the patient knows which medicines to take when they are at home. Tell the patient it is
important to bring their up-to-date list of medicines every time they visit a doctor.
Prevent infection
NPSG.07.01.01
Use the hand cleaning guidelines from the Centers for Disease Control and Prevention or the
World Health Organization. Set goals for improving hand cleaning. Use the goals to improve
hand cleaning.
Prevent mistakes in surgery
UP.01.01.01
Make sure that the correct surgery is done on the correct patient and at the correct place on the
patient's body.
UP.01.02.01
Mark the correct place on the patient's body where the surgery is to be done.
UP.01.03.01
Pause before the surgery to make sure that a mistake is not being made.
Improve staff communication
NPSG.02.03.01
Get important test results to the right staff person on time.
Use alarms safely
NPSG.06.01.01
Make improvements to ensure that alarms on medical equipment are heard and responded to
on time.
Prevent patients from falling
NPSG.09.02.01
Find out which patients and residents are most likely to fall. For example, is the patient or
resident taking any medicines that might make them weak, dizzy or sleepy? Take action to
prevent falls for these patients and residents.
Prevent bed sores
NPSG.14.01.01
Find out which patients and residents are most likely to have bed sores. Take action to prevent
bed sores in these patients and residents. From time to time, re-check patients and residents for
bed sores.
Identify patient safety risks
NPSG.15.01.01
Reduce the risk for suicide.
NPSG.15.02.01
Find out if there are any risks for patients who are getting oxygen. For example, fires in the
patient's home.
NCCT Phlebotomy Review PG5
Detailed Test Plan
Overview
The Detailed Test Plan reflects the results of a national job analysis study that determined the
critical job competencies. Familiarize yourself with each Content Category, Subcategory and
Task on the following pages to complete this section of your review.
View / save a printable Detailed Test Plan

NCCT exam items consist of three different item levels:


● Level 1 - Recall
● Level 2 - Comprehension, Interpretation, Explanation, Application
● Level 3 - Analysis, Problem Solving, Determining Course of Action

Phlebotomy Technician Content Categories:


● Quality and Professional Issues
○ Exam includes 19 scored items from this Category
● Infection Control and Safety
○ Exam includes 22 scored items from this Category
● Orders and Equipment Selection
○ Exam includes 22 scored items from this Category
● Patient ID and Site Preparation
○ Exam includes 23 scored items from this Category
● Collections
○ Exam includes 23 scored items from this Category
● Problems and Correction
○ Exam includes 16 scored items from this Category

Quality and Professional Issues Tasks


Your exam will include 19 scored items related to Quality and Professional Issues
• Perform tasks within the scope of practice (e.g., received in clinical training).
• Perform and record quality control procedures and results (e.g., temperature logs, glucose
meter).
• Identify and follow up with quality control results that do not meet pre-determined criteria (e.g.,
what was corrective action).
• Perform phlebotomy tasks while maintaining patient guarantees under “The Patient Care
Partnership (Patient’s Bill of Rights)” of the American Hospital Association (e.g., consent,
privacy).
• Adapt interactions with patients based on individual needs (e.g., age, culture, special needs).
• Respond to verbal and nonverbal cues when interacting the patients.
• Comply with laws and standards governing specimen collection as related to reliability and
accuracy in lab testing (e.g., CLIA, CAP, COLA, AABB).
• Comply with chain of custody collection requirements (e.g., paternity testing, drug screening,
blood alcohol levels).
• Prevent clerical and technical errors that may occur with specimen collection, handling,
transporting, and processing by following protocol.
• Document patient and collection information electronically or in hard copy format.
• Comply with laws related to medical records and confidentiality (e.g., HIPAA).
• Comply with laws governing reportable incidents (e.g., mistakes, poor patient outcomes).
NCCT Phlebotomy Review PG6
• Monitor quality assurance in the collection of blood specimens.
• Adhere to professional standards of the industry (e.g., hygiene, dress code).

Infection Control and Safety Tasks


Your exam will include 22 scored items related to Infection Control and Safety
• Adhere to regulations regarding work place safety (e.g., OSHA, SDS, NFPA).
• Respond to workplace hazards including fire, electrical, and chemical.
• Take measures to prevent infection and transmission, including hospital acquired infections.
• Follow Standard and Transmission-based Precautions (e.g., airborne, droplet, contact,
hospital-acquired).
• Prevent occurrences that could result in legal action (e.g., hematoma, nerve damage, probing,
patient falls).
• Properly use personal protection equipment including gloves, gown, and masks.
• Use safety products as they are intended (e.g., sharps containers, face shields, blood transfer
device).
• Activate safety mechanisms on phlebotomy equipment appropriately.
• Properly dispose of phlebotomy equipment following OHSA bloodborne pathogens and
hazardous material standards.
• Follow the appropriate course of action for blood and body fluid exposure (e.g., needle stick).
• Perform cleaning and disinfection of equipment and facilities.

Orders and Equipment Selection Tasks


Your exam will include 22 scored items related to Orders and Equipment Selection
• Perform patient registration.
• Perform laboratory test order entry.
• Resolve questionable entries or errors on patient requisition.
• Select appropriate venipuncture equipment for the test ordered and type of patient.
• Select appropriate capillary puncture equipment for the test ordered and type of patient.
• Select proper equipment for patients with allergies.
• Verify quality of equipment (e.g., sterility, recalls, expiration date, defects).
• Select proper antiseptic agents for the test ordered.
• Identify additives/anticoagulants added to evacuated blood collection tubes.
• Assess the mode of action of additives/anticoagulants in blood collection tubes.
• Follow manufacturer recommendations for fill level and tube inversion.
• Select proper bandaging equipment.
• Select appropriate requisitions for specialty lab departments.

Patient ID and Site Preparation Tasks


Your exam will include 23 scored items related to Patient ID and Site Preparation
• Review and clarify orders for patient specimen collection.
• Communicate effectively and professionally with patients (e.g., verbal and non verbal).
• Identify patients according to regulatory standards and facility protocol.
• Evaluate pre-test conditions for patient prior to collection (e.g., fasting, medications, fistula).
• Assess pre-analytical practices that can affect results positively or negatively (e.g., heating
pads, fist pumping).
• Select the appropriate site for venous blood collection.
• Select the appropriate site for arterial blood collection.
• Select the appropriate site for capillary blood collection.
• Apply and release the tourniquet appropriately.
• Prepare the site for blood collection based on test ordered (e.g., venipuncture in antecubital for
CBC with alcohol vs blood cultures with chlorhexidine).
NCCT Phlebotomy Review PG7
Collections Tasks
Your exam will include 23 scored items related to Collections
• Prioritize patient collections based on order request (e.g., STAT, timed, routine).
• Take precautions for patients with special needs (e.g., breast surgery, IV, burns, dementia,
bleeding disourders).
• Recognize commonly ordered tests and the tubes needed for collection.
• Perform special collections (e.g., trace metal elements, newborn screen, chain of custody).
• Properly anchor the vein.
• Position the needle for venipuncture (e.g., direction, angle, depth).
• Collect samples using CLSI recommended order of draw.
• Perform appropriate post-puncture care for the patient.
• Label specimens for facility protocol.
• Deliver specimens to the laboratory appropriately (e.g., temperature, light, time).
• Process specimens for the laboratory appropriately (e.g., centrifuge, aliquot, storage).
• Assess the suitability of a specimen for analysis.
• Deliver specimens to the correct department in the clinical laboratory.
• Perform blood culture collection.
• Perform capillary puncture collection.
• Instruct the patient in the proper collection and preservation of laboratory specimens (e.g.,
blood, sputum, urines, stools).
• Collect laboratory specimens per requirements (e.g., urine, stool, culture swabs).
• Process standard non-blood specimens (e.g., urine, sputum, stool, swabs).
• Process special non-blood specimens (e.g., CSF, synovial fluid, peritoneal fluid, pericardial
fluid).
• Process blood specimens for testing at reference laboratories.
• Package blood specimens for transport to reference laboratories per protocol.
• Arrange for transport of blood specimens for testing at reference laboratories.
• Perform point of care testing (e.g., urinalysis, hemoglobin and hematocrit, coagulation,
glucose, pregnancy test, occult blood).
• Report results and critical values to authorized personnel.

Problems and Correction Tasks


Your exam will include 16 scored items related to Problems and Correction
• Prevent interference in clinical analysis of blood constituents (e.g., iodine, alcohol, edema, IV
fluids).
• Prevent pre-analytical sources of error regarding specimen integrity (e.g., hemolysis, QNS,
clotted, incorrect specimen type).
• Perform specimen collection on difficult to draw patient (e.g., chemotherapy, dialysis, edema,
pediatric, geriatric, dehydration, obesity) using appropriate techniques.
• Take appropriate action when blood return is not established (e.g., collapsed vein, missed
vein).
• Respond to patient adverse reactions that may accompany blood collection (e.g., hematoma,
petechiae, nerve injury, diaphoresis, syncope, nausea, seizure).
• Make phlebotomy-related decisions for patients on anticoagulant therapy or with clotting
deficiencies (e.g., hold pressure for longer period of time following collection).
• Take corrective actions for problems with test requests, specimen transport, or specimen
processing.
• Take corrective actions for misidentified patients or samples.
• Take corrective actions for patient complaints or refusals.
NCCT Phlebotomy Review PG8
Certification Examination Questions
General Information & Sample Test Items
All NCCT certification examinations contain questions written at three levels:
● Level 1 (recall of information)
● Level 2 (understanding or applying information)
● Level 3 (analyzing information or making judgments/decisions)
The following are examples of questions at each level. They have been chosen from different
content categories in different healthcare disciplines. Actual certification exam questions will be
discipline specific.

Sample Question, Level 1:


For insurance processing purposes, what does the abbreviation CPT mean?
● A. Current Procedural Terminology
● B. Codes and Procedural Terms
● C. Current Processing Terminology
● D. Codes and Processing Terms
Explanation of level: This question asks the examinee to recall a fact.

Sample Question, Level 2:


If a patient tells the Patient Care Technician (PCT) that he does not want to get out of bed and
the PCT tries to move him anyway, the PCT could be charged with __________.
● A. gross misconduct
● B. negligence
● C. assault
● D. battery
Explanation of level: This question requires the examinee to interpret the action of the PCT in
relationship to the definition of the legal terms provided, which the examinee must also know.
Interpretation indicates understanding.

Sample Question, Level 3:


A Phlebotomy Technician is asked to make a 10% solution of chlorine bleach to use for cleaning
the countertops. To make a total volume of one liter (L) of this 10% solution, which of these
combinations is the best choice?
● A. 900 mL chlorine bleach and 100 mL water
● B. 1 L chlorine bleach and 10 mL water
● C. 100 mL chlorine bleach and 900 mL water
● D. 10 mL chlorine bleach and 1 L water
Explanation of level: This question asks the examinee to recall and apply mathematical “rules”
for making solutions. Examinee must also know how the phase of matter (i.e., solid, liquid, gas)
for each component affects interpretation of the rules in order to make a decision. It requires
complex thinking skills.
NCCT Phlebotomy Review PG9
VOCAB - Common Tests (105 Words)
1. MRSA
○ Methicillin-resistant Staphylococcus aureus
2. BBP
○ Bloodborne Pathogen
3. BAC
○ Blood Alcohol Concentration
4. aPTT
○ Activated Partial Thromboplastin Time
5. FBS
○ Fasting Blood Sugar
6. pH
○ Hydrogen ion concentration (measure of acidity or alkalinity)
7. TSH
○ Thyroid Stimulating Hormone
8. RIA
○ Radioimmunoassay
9. Crit
○ Hematocrit
10. ASO
○ Antistreptolysin O
11. BUN
○ Blood Urea Nitrogen
12. DIC
○ Disseminated intravascular coagulation
13. HPV
○ Human Papillomavirus
14. PSA
○ Prostate Specific Antigen
15. TB
○ Tuberculosis
16. MCV
○ Mean Corpuscular volume
17. CBC
○ Complete Blood Count (Laboratory Test)
18. GTT
○ Glucose Tolerance Test
19. T3
○ Triiodothyronine
20. HDL
○ High-Density Lipoprotein (a.k.a. “Goog” cholesterol)
NCCT Phlebotomy Review PG10
21. XDP
○ Crosslinked (Fibrinogen) Degradation Product
22. FDP
○ Crosslinked (Fibrinogen) Degradation Product
23. LH
○ Luteinizing Hormone
24. CK
○ Creatine Kinase
25. DNA
○ Deoxyribonucleic Acid
26. ESR
○ Erythrocyte Sedimentation Rate
27. UV
○ Ultraviolet
28. ACTH
○ Adrenocorticotropic Hormone
29. T4
○ Thyroxine
30. Fe
○ Iron
31. Lytes
○ Electrolytes
32. MCHC
○ Mean corpuscular hemoglobin concentration
33. WBC
○ White Blood Cells (Part of a CBC - Complete Blood Count)
34. CHOL
○ Cholesterol
35. VIT A
○ Vitamin A, Retinol
36. SLE
○ Systemic lupus erythematosus
37. ALP
○ Alkaline Phosphatase
38. K+
○ Potassium
39. ADH
○ Antidiuretic Hormone
40. Polys
○ Polymorphonuclear Leukocytes
41. Gluc
○ Glucose
42. LD
○ Lactic Dehydrogenase
NCCT Phlebotomy Review PG11
43. LDH
○ Lactic Dehydrogenase
44. VDRL
○ Venereal Disease Research Lab.
45. VD
○ Venereal Disease
46. EBV
○ Epstein-Barr Virus
47. BC or B/C
○ Blood Culture
48. TDM
○ Therapeutic Drug Monitoring
49. HBV
○ Hepatitis B Virus
50. CSF
○ Cerebrospinal Fluid
51. Ca
○ Calcium
52. Path
○ Pathology
53. HCV
○ Hepatitis C Virus
54. O2
○ Oxygen
55. Bili
○ Bilirubin (Protect Specimen From Light)
56. ANA
○ Antinuclear Antibody
57. EtOH
○ Chemical Abbreviation for ethanol (commonly blood alcohol)
58. AST
○ Aspartate Aminotransferase
59. Plt (PLT)
○ Platelets (Part of a CBC - Complete Blood Count)
60. BT
○ Bleeding Time
61. CO2
○ Carbon Dioxide
62. ABG
○ Arterial Blood Gasses
63. HbsAg
○ Hepatitis B Surface Antigen
64. INR
○ International Normalized Ratio (Standardized unit for prothrombin time results)
NCCT Phlebotomy Review PG12
65. MCH
○ Mean Corpuscular Hemoglobin
66. Staph
○ Staphylococcus
67. PCO2
○ 25 - Hydroxy Vit D
68. TIBC
○ Total Iron Binding Capacity
69. Na or Na+
○ Sodium
70. FDP
○ Fibrinogen or Fibrin Degradation Product
71. RPR
○ Rapid Plasma Reagin
72. HCG
○ Human Chorionic Gonadotropin
73. HgbA1c
○ Hemoglobin A1c (Glycosylated Hemoglobin)
74. Segs
○ Segmented white blood cells
75. Eos
○ Eosinophils
76. HIV
○ Human Immunodeficiency Virus
77. AST
○ Aspartate Aminotransferase
78. ABO
○ Major Blood Group System (Blood Type: A, B, AB, and O)
79. Sed Rate
○ Erythrocyte Sedimentation Rate (ESR)
80. Hgb, Hb
○ Hemoglobin (Part of a CBC - Complete Blood Count)
81. O&P
○ Ova and Parasite
82. FSH
○ Follicle Stimulating Hormone
83. HSV
○ Herpes Simplex Virus
84. HDL
○ Low-Density Lipoprotein (a.k.a. “Bad” cholesterol)
85. UA, ua
○ Urinalysis
86. Lymphs
○ Lymphocytes
NCCT Phlebotomy Review PG13
87. Strep
○ Streptococcus
88. POCT
○ Point of Care Testing
89. APTT
○ Activated Partial Thromboplastin Time
90. ALT
○ Alanine Transaminase
91. Diff
○ Differential White Count (Part of a CBC - Complete Blood Count)
92. ALT
○ Alanine Aminotransferase
93. PPD
○ Purified Protein Derivative (TB Test)
94. STD
○ Sexually Transmitted Disease
95. T Cells
○ Lymphocytes from the thymus
96. Hct
○ Hematocrit (Part of a CBC - Complete Blood Count)
97. PO2
○ Pressure of oxygen in the blood
98. Mg
○ Magnesium
99. PT
○ Pro-Time (Prothrombin TIme)
100. TRIG
○ Triglycerides
101. T&C
○ Type and crossmatch (type & x)
102. PKU
○ Phenylketonuria
103. PTH
○ Parathyroid Hormone
104. PMNs
○ Polymorphonuclear Leukocytes
105. RBC
○ Red Blood Cells (Part of a CBC - Complete Blood Count)
NCCT Phlebotomy Review PG14
VOCAB -Terminology (119 Words)
1. P.o.
○ Orally (per os)
2. ID
○ Intradermal
3. Req
○ Requisition
4. H2O
○ Water
5. Wd
○ Wound
6. Hypoxia
○ Low or Deficient Oxygen
7. Mm
○ Millimeter (ex. 3mm superficial laceration)
8. Dx
○ Diagnosis
9. Tx
○ Treatment
10. Mets
○ Metastases
11. ASAP
○ As Soon As Possible (This term is similar to STAT)
12. Hematoma
○ Swelling of clotted blood in the tissues (Can result from a venipuncture)
13. Sepsis / Septicemia
○ Bloodstream Infection
14. Yrs or yrs. (y/o or y.o. )
○ Years (Years old)
15. Lymphedema
○ Swelling of the lymphatics
16. Anemia
○ Blood condition where there is a lack of red blood cells
17. PRN, prn
○ As Necessary (pre renata)
18. COPD
○ Chronic Obstructive Pulmonary Disease
19. ER / ED
○ Emergency Room / Emergency Department
20. AML
○ Acute Myelocytic Leukemia
21. Fibrogen
○ Protein in plasma that is converter to fibrin by thrombin
NCCT Phlebotomy Review PG15
22. TPN
○ Total Parenteral Nutrition (Intravenous Feeding)
23. CPR
○ Cardiopulmonary Resuscitation
24. Sol
○ Solution
25. Erythrocytosis
○ Increase in red cell mass and hematocrit
26. AV Fistula
○ Arterio - Venous FIstula (also known as a shunt)
27. FUO
○ Fever of unknown origin
28. ENT
○ Ear, Nose and Throat
29. C-Section
○ Cesarean Section
30. Prep
○ Prepare for
31. Lymphostasis
○ Obstruction of flow of lymph
32. CL
○ Chloride
33. Thrombosis
○ Blood Clot within the vascular system
34. WT, wt
○ Weight
35. CNS
○ Central Nervous System
36. Vasculitis
○ Small Vessel Inflammation
37. DNR
○ Do Not Resuscitate
38. Hx
○ History
39. Mg
○ Milligram
40. PP
○ PostPrandial (Latin for after a meal.)
41. OR
○ Operating Room
42. Hematology
○ The study of blood
NCCT Phlebotomy Review PG16
43. AIDS
○ Acquired Immune Deficiency Syndrome ( or Acquired Immunodeficiency
Syndrome)
44. Neg
○ Negative
45. Embolus
○ A blood clot carried in the bloodstream
46. Chemo
○ Chemotherapy
47. Lymphoma
○ Lymph mass or fluid collection (common cancer involving lymphocytes)
48. Pre-op
○ Before Operation
49. IV
○ Intravenous
50. Polycythemia
○ Condition of increase in red blood cells, white blood cells and platelets
51. ALL
○ Acute Lymphocytic Leukemia
52. Bx
○ Biopsy
53. DOB
○ Date of Birth
54. Oz.
○ Ounce
55. w/o
○ Without (the symbol “s” with a line above it also indicates “without” S¯ )
56. Coagulation
○ Clotting (Process by which blood changes from liquid to gel/clot)
57. Lymphangitis
○ Lymph Vessel Inflammation
58. X
○ Times
59. Hyperglycemia
○ Excessive sugar in the blood condition
60. Sx
○ Symptoms
61. ETS or EST
○ Evacuated Tube System
62. Ml
○ Milliliter
63. SPS
○ Sodium Polyanethol Sulfonate
NCCT Phlebotomy Review PG17
64. PICC
○ Peripherally Inserted Central Catheter
65. ACD
○ Acid Citrate Dextrose
66. T
○ Temperature
67. H, hr
○ Hour
68. DVT
○ Deep vein Thrombosis
69. Ad lib
○ As desired
70. MS
○ Multiple Sclerosis
71. Aq
○ Water
72. Erythropoietin
○ Hormone that increases production of red blood cells
73. PE
○ Pulmonary Embolism
74. P.c.
○ After meals
75. Aggulation
○ Clumping process
76. MRI
○ Magnetic Resonance Imaging
77. Kg
○ Kilogram
78. OB
○ Obstetrics
79. Splenomegaly
○ Spleen Enlargement
80. Lymph
○ Clear colorless liquid found in the lymphatic system
81. Lymphadenitis
○ Lymph Gland Inflammation
82. C¯ / “C” with a line directly above
○ Symbol for “with” (Latin: Cum)
83. CML
○ Chronic Myelogenous Leukemia
84. Anaphylaxis
○ Strong Allergic Reaction
85. Mg++
○ Magnesium
NCCT Phlebotomy Review PG18
86. NPO or n.p.o.
○ Latin for “nil per os” (Nothing By Mouth)
87. Erythropoiesis
○ Means “red, cell, formation”
88. NG
○ Nasogastric
89. AED
○ Automated External Defibrillator
90. GI
○ Gastrointestinal
91. PICC Line
○ Peripherally inserted central catheter line
92. STAT, stat
○ Immediately (Statum)
93. Y.o. or y/o
○ Years Old
94. Anticoagulant
○ Means “Against clots forming”
95. Mono
○ Monocyte
96. Autotransfusion
○ “Self, across, pour, process” (The process of infusing a patient's own blood)
97. Anisocytosis
○ Unequal size and shape of RBCs
98. CAD
○ Coronary Artery Disease
99. R/O
○ Rule Out
100. Pancytopenia
○ “All cell lack of” or lack of all cellular elements of the blood
101. EDTA
○ Ethylenediaminetetraacetic acid
102. Dil.
○ Dilute
103. Post-Op
○ After Operation
104. Leukapheresis
○ White blood cell removal
105. Hemolysis
○ Destruction (lysis) of blood cells
106. Peds
○ Pediatrics
107. Petechiae
○ Hemorrhagic Spots
NCCT Phlebotomy Review PG19
108. Chem
○ Chemistry
109. SST
○ Serum Separator Tube
110. h/o
○ History Of
111. UTI
○ Urinary Tract Infection
112. Im
○ IntraMuscular (regarding injections)
113. AV Shunt
○ Arterio-Venous Shunt (also known as a fistula)
114. PST
○ Plasma Separator Tube
115. QNS, q.n.s.
○ Quantity Not Sufficient
116. Plasmapheresis
○ Removal of plasma from the blood
117. Rx
○ Prescription
118. MI
○ Myocardial infarction (ex. Heart attack)
119. ICU
○ Intensive Care Unit
NCCT Phlebotomy Review PG20
Preliminary Exam - 150Q’s
1. The phlebotomist is preparing to remove her gloves after a venipuncture and notices blood on
the gloves. Where should the phlebotomist dispose of her gloves?
A. Specimen transport bag
B. Regular trash container
C. Red sharps container
D. Biohazard waste container

2. Which of the following steps is completed FIRST in performing venipuncture?


A. Cleanse site with an alcohol wipe
B. Identify the patient
C. Apply the tourniquet
D. Arrange tubes in order of draw

3. Which of the following is the correct order of draw for a STAT lactic acid, lead level, and
calcium?
A. SST, royal blue, gray
B. royal blue, gray, SST
C. royal blue, SST, gray
D. gray, royal blue, SST

4. The physician orders a prothrombin time. Which of the following evacuated tubes should the
phlebotomist select for collecting the specimen?
A. gray
B. light green
C. red
D. light blue

5. Instead of obtaining consent from a patient who is mentally competent, the phlebotomist
obtains consent from a family member. This is a violation of
A. the Patient's Bill of Rights.
B. the Stark Law.
C. HIPAA.
D. CLIA Waived Testing.

6. A phlebotomist has active symptoms of the common cold. Under which of the following
conditions may he perform venipuncture?
A. The phlebotomist may perform venipuncture provided the patient is wearing a mask.
B. The phlebotomist may not perform venipuncture with an active cold.
C. The phlebotomist may perform routine venipuncture as usual.
D. The phlebotomist may perform venipuncture provided he is wearing a mask and afebrile.
NCCT Phlebotomy Review PG21
7. A patient on coumadin therapy is at the lab for a weekly coagulation test. Which of the
following tubes should the phlebotomist collect for this test?
A. Light blue top
B. Red top
C. Lavender top
D. Green top

8. When a phlebotomist enters a patient’s room to collect a STAT blood sample, a sign above
the bed states that all collections should be from the patient’s central port. Which of the
following should the phlebotomist do NEXT?
A. Communicate the test order to the nursing staff and wait for one of them to collect the
sample.
B. Identify the patient and ask for confirmation that the sign is accurate before proceeding with
the collection.
C. Proceed with sample collection from the central port and alert the nurse when finished.
D. Exit the patient’s room and proceed directly to the next patient on the collection list.

9. Which of the following actions should the phlebotomist take to decrease the potential for
hemolysis of a blood specimen in an EDTA tube?
A. Use a 25 gauge needle or smaller to collect the specimen.
B. Store the tube in the refrigerator after the specimen has been collected.
C. Gently invert the tube after collecting the specimen.
D. Allow the tube to stand for 20 minutes prior to processing.

10. An inpatient with a left-sided mastectomy has a hematoma in the left arm and an active IV in
the right arm. Which of the following is an appropriate site for venipuncture?
A. Proximal to the hematoma
B. Distal to the hematoma
C. Distal to the IV
D. Proximal to the IV

11. The phlebotomist needs to draw a blood culture, sedimentation rate, PTT, and a glucose
test. Which of the following is the correct order of draw?
A. Gray, light blue, lavender, red
B. Yellow, light blue, lavender, gray
C. Red, yellow, light blue, lavender
D. Yellow, red, lavender, gray
NCCT Phlebotomy Review PG22
12. Which of the following forms is the phlebotomist responsible for having the patient sign
when a specimen is being collected for substance abuse screening for employment?
A. informed consent
B. chain-of-custody
C. release of medical Information
D. advance directive

13. After performing a venipuncture in a hospital setting and activating the safety device on the
needle, which of the following is the correct method of disposing of the used needles, syringe,
and ETS?
A. Throw the used needle into a sharps container and the syringe and ETS into a biohazard bin.
B. Throw the entire assembly into the bedside trash.
C. Throw the entire assembly into a sharps container.
D. Throw the entire assembly into a biohazard bin.

14. Which of the following actions is a HIPAA violation?


A. Shredding completed collection lists
B. Disposing of extra patient tube labels in a regular trash can
C. Putting a note in the laboratory reminding staff that a certain patient has a cold agglutinin
D. Storing a fully stocked collection tray in the clinical laboratory

15. Blood cultures need to be drawn from a patient who is sensitive to iodine. Which of the
following antiseptics should be used to cleanse the collection site?
A. 70% isopropyl alcohol
B. 2% povidone
C. 0.5% chlorhexidine
D. 2% hydrogen peroxide

16. Is it appropriate for the phlebotomist to use an alcohol-based hand rub as the only method
of hand hygiene after which of the following activities?
A. Before putting on gloves prior to venipuncture
B. After removing soiled gloves
C. Before entering the employee break room
D. After using the restroom

17. While performing a venipuncture on an obese patient, the appropriate angle of insertion is
A. 20-35 degrees.
B. 10-25 degrees.
C. 15-30 degrees.
D. 30-45 degrees.
NCCT Phlebotomy Review PG23
18. Which of the following quality control measures is a phlebotomist responsible for? (Select
the three (3) correct answers.)
A. checking expiration dates
B. running controls on POC testing
C. calibrating lab equipment
D. recording abnormal test results
E. recording refrigerator temperatures

19. Which of the following is the correct order of PPE removal prior to leaving the exam room of
a patient who is in contact-droplet isolation?
A. Gloves, mask, gown
B. Gown, mask, gloves
C. Gown, gloves, mask
D. Gloves, gown, mask

20. Which of the following BEST describes the proper way to clean up a broken glass tube?
A. Use a broom and dustpan to sweep up the glass and fragments and place in a sharps
container.
B. Use a paper towel to pick up the pieces of glass and place them in a trash can.
C. Use a wet towel and wipe over the area so that minute fragments of glass may be picked up.
D. Place the pieces of glass on a piece of paper and carry the paper to the red biohazard bag.

21. When selecting a lancet for collecting blood from the heel of a newborn, the length should
be
A. 2.4 mm or greater
B. 2.4 cm or greater
C. 2.0 mm or less
D. 2.0 cm or less

22. The phlebotomist enters a non-English speaking patient’s room with her supplies. The
patient extends his arm. In this case the patient has
A. granted implied consent.
B. granted informed consent.
C. granted expressed consent.
D. not legally granted consent.

23. The phlebotomist applies the tourniquet, but then has trouble relocating the vein after
several minutes of palpation. Which of the following is an expected outcome in these test
results?
A. Falsely elevated K levels from tissue damage
B. Falsely increased platelets from clotting
C. Falsely decreased iron levels from hemolysis
D. Fibrinolysis from hemodilution
NCCT Phlebotomy Review PG24
24. Which of the following equipment is MOST appropriate for drawing a CBC from a 5-year-old
patient?
A. 23 g Vacutainer and a lavender top tube
B. 23 g Vacutainer and a light blue top tube
C. 25 g Vacutainer and a lavender top tube
D. 25 g Vacutainer and a light blue top tube

25. During palpation of a large vessel in the forearm, the phlebotomist detects a vibration
sensation. The phlebotomist should then
A. use a syringe blood collection set up with minimal plunger pressure.
B. use a lower angle of penetration with a butterfly.
C. select a smaller gauge needle for the draw site.
D. choose a different collection site.

26. The patient reports feeling faint and slumps forward as soon as the needle is inserted into
the arm during a venipuncture. Which of the following actions should the phlebotomist take
next?
A. Use smelling salts to revive the patient.
B. Remove the needle and tourniquet.
C. Continue obtaining the specimen as quickly as possible.
D. Apply a cold compress to the back of the patient’s neck.

27. The phlebotomist is asked to draw blood on a non-responsive patient in an emergency


department. This draw would be conducted under
A. Refusal of consent.
B. informed consent.
C. expressed consent.
D. implied consent.

28. Forceful shaking of blood in an evacuated collection tube will cause which of the following
complications?
A. Hemolysis
B. Hemoconcentration
C. Contamination
D. Hemostasis

29. A phlebotomist has received a requisition to collect specimens for a Bilirubin from a
premature newborn patient. Which of the following is the BEST device for this collection?
A. 1.00 mm lancet
B. Winged infusion set
C. Syringe and needle
D. 3.00 mm lancet
NCCT Phlebotomy Review PG25
30. Which of the following additional PPE should the phlebotomist wear when entering a room
marked for airborne precautions?
A. face shield
B. mask
C. non-latex gloves
D. N95 Respirator

31. Which of the following is the CLSI recommended micro-collection order of draw for a BMP,
CBC and bilirubin?
A. Green, lavender, red
B. Lavender, red, green
C. Lavender, green, red
D. Red, green, lavender

32. Which of the following tube and additive combinations should be used for a PT/INR test?
A. Light blue, sodium citrate
B. Light blue, sodium heparin
C. Gray, sodium citrate
D. Gray, sodium heparin

33. A 2-week-old infant has been brought in the lab for a repeat PKU/neonatal screening. Which
of the following collection procedures is appropriate to collect the specimens?
A. venipuncture using evacuated tube system
B. capillary using heel puncture lancet
C. capillary using dermal puncture lancet
D. venipuncture using winged infusion set

34. Which of the following actions by the phlebotomist is the FIRST line of defense in preventing
the spread of microorganisms?
A. Wear non-sterile gloves when performing venipuncture
B. Use aseptic technique when drawing up a medication
C. Wear sterile gloves when changing a dressing
D. Perform regular hand hygiene

35. Which of the following additives prevents glycolysis?


A. potassium oxalate
B. EDTA
C. sodium fluoride
D. sodium citrate
NCCT Phlebotomy Review PG26
36. When preparing a site with alcohol for venipuncture, the phlebotomist should cleanse in
which of the following motions?
A. horizontal or vertical back-and-forth friction
B. concentric from the periphery to the center
C. light horizontal strokes
D. very light upward strokes

37. Which of the following actions should the phlebotomist take to make a vein more prominent
when attempting to select a venipuncture site?
A. Apply a warm compress to the area for 5 minutes.
B. Apply firm pressure to the arm from the shoulder to the elbow.
C. Leave the tourniquet in place and ask the patient to dangle the arm for one minute.
D. Firmly tap the arm several times and palpate with a non-gloved finger

38. The phlebotomist applies pressure to the venipuncture site using gauze. After two minutes
he observes that bleeding has not stopped. The NEXT appropriate action would be to
A. wrap the arm with a pressure dressing.
B. notify the physician immediately.
C. continue to apply pressure while bending the arm at the elbow.
D. elevate the arm while applying pressure.

39. Which of the following precautions should be used when drawing blood on patients
diagnosed with tuberculosis?
A. Standard and Contact Precautions
B. Standard Precautions
C. Universal Precautions
D. Standard and Airborne Precautions

40. Keeping contaminated equipment and supplies away from the phlebotomist’s clothing to
prevent pathogen transmission to the next patient is an example of which of the following?
A. Surgical asepsis
B. Medical asepsis
C. Sanitization
D. Disinfection

41. A phlebotomist puts evacuated tubes filled with blood in plastic transport bags and places
them in the physician office lock box for the reference lab courier who will pick them up in 1-2
hours. The lock box is outside and the temperatures are below freezing. How will freezing
affect the test results?
A. Elevated hemoglobin
B. Elevated hematocrit
C. Decreased calcium
D. Decreased red blood cell count
NCCT Phlebotomy Review PG27
42. Which of the following regulations set the minimum standards for medical laboratory practice
and quality?
A. HIPAA
B. DEA
C. CLIA
D. OSHA

43. The phlebotomist is performing a venipuncture on an apprehensive school age child. Which
of the following approaches should the phlebotomist use to calm the child?
A. Direct the child’s questions to the parent and have the parent explain.
B. Allow the child to clean the puncture site and give her a sticker.
C. Explain the procedure and reassure the child that the parent can stay with her.
D. Tell the child she has to do this, but it won’t hurt.

44. A test tube with which of the following stopper colors prevents the conversion from
prothrombin to thrombin?
A. yellow
B. red
C. green
D. lavender

45. The patient on hemodialysis with a left AV shunt has undergone a right sided mastectomy
and has an order for a STAT glucose. After consulting with the physician the phlebotomist is
told to perform a capillary draw. Which of the following sites should the phlebotomist use?
A. Left ring finger
B. Right little finger
C. Left thumb
D. Right middle finger

46. Small red spots appear near the tourniquet on a patient's arm. This is MOST likely an
indication of
A. a hematoma.
B. edema.
C. petechiae.
D. a collapsed vein.

47. Which of the following is the most common light sensitive analyte specimen?
A. calcium
B. potassium
C. bilirubin
D. folic acid
NCCT Phlebotomy Review PG28
48. When performing a capillary puncture on an infant, which of the following is the reason for
using the lateral heel instead of a finger?
A. The blood in the fingertip has been found to contain a higher concentration of hemoglobin
than the heel.
B. The finger produces a slower flow of blood, making it more difficult to obtain a specimen.
C. The tissue layers on the finger are more susceptible to permanent nerve damage.
D. The small amount of tissue between the skin and bone in the finger makes an injury to the
bone likely.

49. Which of the following conditions may occur if a tourniquet is left in place on a patient’s
extremity for longer than 60 seconds?
A. Venous insufficiency
B. Hemoconcentration
C. Hemangioma
D. Fibrinolysis

50. The phlebotomist notes the patient's arm is scarred in the lateral aspect of the antecubital
fossa, and the other arm is not available. The phlebotomist decides to use a more medial site in
the fossa over the basilic vein. Which of the following make this a poor choice? (Select the two
(2) correct answers.)
A. Increased risk of nerve damage
B. Increased risk of hemolysis
C. Increased risk of an unsuccessful draw
D. Increased risk of arterial involvement
E. Increased risk of specimen rejection

51. Which of the following is a requirement for a healthcare facility to be in compliance with
OSHA?
A. Current certificate of compliance on file
B. Annual inspections conducted by OSHA
C. Payment of annual dues
D. Annual employee safety training

52. While giving a patient an injection, the patient jumped, causing the phlebotomist to get stuck
on the hand with a contaminated needle. After performing thorough hand washing, which of the
following should the phlebotomist do FIRST?
A. Complete an exposure incident report.
B. Begin infectious disease prophylaxis.
C. Report the incident to a supervisor.
D. Obtain patient consent for infectious disease testing.
NCCT Phlebotomy Review PG29
53. A two-year-old toddler was brought into the lab for Hct and Hgb. Which of the following sites
should be used for the dermal puncture?
A. Right heel
B. Index finger of non-dominant hand
C. Middle finger of non-dominant hand
D. Left great toe

54. Which of the following veins would generally be acceptable for routine blood collection by
the phlebotomist?
A. Cephalic
B. Femoral
C. Subclavian
D. Dorsalis pedis

55. Which of the following is the reason blood cultures are typically drawn first?
A. They can easily be contaminated.
B. A greater quantity of blood is required.
C. They take more time to collect.
D. The blood needs to be in contact with the additive longer.

56. When inspecting a patient’s arm before performing a venipuncture, the most desirable site
appears to be the back of the hand. Which of the following venipuncture methods is MOST
appropriate in this situation?
A. Capillary puncture
B. Butterfly needle
C. Evacuated tube
D. Needle and syringe

57. A physician has ordered STAT hematology, chemistry, and coagulation tests on a patient.
Which of the following tubes are needed for these procedures? (Select the three (3) correct
answers.)
A. Light blue
B. Lavender
C. Gray
D. Green
E. Yellow

58. A phlebotomist is using an evacuated tube holder and multi-sample needle to collect a CBC,
PT, and BUN. The correct order of draw for this collection is
A. lavender, light blue, green.
B. green, light blue, lavender.
C. green, lavender, light blue.
D. light blue, green, lavender.
NCCT Phlebotomy Review PG30
59. Which of the following should the phlebotomist do after drawing blood from a patient on
anticoagulant therapy? (Select the three (3) correct answers.)
A. Have the patient bend the arm and hold pressure.
B. Call the nurse for instructions.
C. Wait for bleeding to stop.
D. Apply a pressure dressing over the site.
E. Hold pressure and elevate the patient's arm.

60. If a Spanish-speaking patient is not provided a translator, although one was requested,
would be considered a violation of
A. Health Insurance Portability and Accountability Act.
B. protected health information.
C. The Patient’s Bill of Rights.
D. patient confidentiality.

61. During a blood draw, a patient begins complaining of discomfort and tingling, at which time
the phlebotomist notices bruising and discoloration at the site. How should the phlebotomist
proceed?
A. Release the tourniquet and complete the draw.
B. Release the tourniquet, withdraw the needle, and apply pressure.
C. Complete the draw, release the tourniquet, and apply pressure.
D. Withdraw the needle immediately, apply pressure, and apply ice.

62. Which of the following tube stopper colors is used to collect blood cell counts?
A. red/gray speckled top
B. green
C. lavender
D. red

63. If a phlebotomist has questions about disposing of outdated hazardous chemicals, where
should she look to find this information?
A. HIPAA
B. NIOSH
C. SDS
D. CLIA

64. At which of the following times should a test tube be labeled with the patient’s identification?
A. After leaving the patient’s room
B. While in the patient’s room, after drawing the blood
C. While in the patient’s room, before drawing the blood
D. Before entering the patient’s room
NCCT Phlebotomy Review PG31
65. The Needlestick Safety and Prevention Act exists to protect healthcare workers from
accidental exposure to
A. carcinogens.
B. blood borne pathogens.
C. biologic toxins.
D. hazardous chemicals.

66. While obtaining blood during a venipuncture, the patient faints. Which of the following
actions should the phlebotomist take initially?
A. Reposition the needle slightly and obtain the ordered specimen.
B. Discontinue the venipuncture and call for help.
C. Call the patient’s emergency contact on file.
D. Apply direct pressure for five full minutes on the venipuncture site.

67. A phlebotomist must collect type and screen, STAT electrolytes, D-dimer, and estrogen.
Which of the following correctly lists the appropriate additives for these tests in the CLSI
recommended order of draw?
A. Non-additive, sodium citrate, lithium heparin, potassium EDTA
B. Silica, sodium citrate, sodium heparin, potassium EDTA
C. Sodium citrate, non-additive, lithium heparin, potassium EDTA
D. Sodium citrate, silica, sodium heparin, potassium EDTA

68. While performing a venipuncture using an evacuated tube, a small amount of blood enters
the tube and then stops. Which of the following conditions is MOST likely to have caused this?
A. Hematoma
B. Collapsing vein
C. Rolling vein
D. Sclerosed vein

69. Which of the following techniques should the phlebotomist use when performing a
venipuncture on a geriatric patient with loose skin?
A. Vigorous massaging of the arm to increase blood flow in the veins
B. Pulling the skin taut using gentle and sufficient pressure to anchor the vein
C. Using a sharper angle and slower needle insertion to decrease rolling
D. A tighter tourniquet to anchor more fragile veins

70. When performing a venipuncture, sudden swelling is observed at the insertion site. Which
of the following actions should the phlebotomist take?
A. Continue obtaining the blood sample. This is a localized reaction to the venipuncture.
B. Apply firm pressure above the area of swelling to direct the blood flow into the collection
device.
C. Release the tourniquet, remove the needle, and immediately apply pressure at the site.
D. Leave the tourniquet in place, immediately remove the needle, and apply pressure at the site.
NCCT Phlebotomy Review PG32
71. Which of the following personal wellness areas conveys the strongest impression of the
optimal health of an individual?
A. Exercise
B. Personal Hygiene
C. Stress Management
D. Nutrition

72. A phlebotomist must collect a specimen for a blood alcohol test on a patient who is allergic
to shellfish. Which of the following antiseptics should the phlebotomist use?
A. 70% isopropyl alcohol
B. Benzalkonium chloride
C. Chlorhexidine gluconate
D. Iodine

73. A 17-month-old presents for a routine capillary stick to determine lead levels. The point of
draw should be the
A. finger.
B. thumb.
C. toe.
D. heel.

74. The physician has ordered a prothrombin time (PT) and complete blood count (CBC) for an
adult patient with a diagnosis of thrombophlebitis. The phlebotomist should collect the
evacuated tubes in which of the following orders?
A. Lavender, light blue
B. Light blue, lavender
C. Light blue, red
D. Red, light blue

75. Which of the following is an appropriate draw site for an adult patient with a central line?
A. Dorsal side of the hand
B. Plantar surface of the heel
C. Palmar side of the wrist
D. Distal phalanx of the fifth finger

76. A phlebotomist completed venipuncture on a patient for bilirubin and carotene. Which of the
following must the phlebotomist do with these lab tests to prevent altering the chemical
composition?
A. Invert the test tube 10 times and take it directly to the lab.
B. Keep the test tube away from direct light by wrapping it in foil.
C. Place the test tube in an ice slurry.
D. Invert the test tube 10 times and set it on the testing rack.
NCCT Phlebotomy Review PG33
77. Which of the following POC laboratory tests is within the CLIA defined scope of practice for
a phlebotomist?
A. Cell differential and urine sediment
B. Urine dip and cell differential
C. Strep test and urine sediment
D. Strep test and urine reagent strip

78. A patient with an order for a blood draw presents with burns covering both arms. Which of
the following is the phlebotomist’s BEST option for collecting the largest amount of blood for this
patient?
A. Filter paper (PKU) card
B. Evacuated tube
C. Microcollection container
D. Capillary tube

79. A phlebotomist collects a cryofibrinogen specimen, places the specimen in a biohazard


specimen bag, then onto a slurry of ice and water and transports it to the laboratory in a timely
manner. In this case the phlebotomist
A. should have protected the specimen from light.
B. should have kept the specimen at 37˚C.
C. performed proper specimen handling.
D. should have aliquoted the specimen before transport.

80. In the middle of a blood draw the patient experiences a seizure. Which of the following
should the phlebotomist do after removing the tourniquet and withdrawing the needle? (Select
the two (2) correct answers.)
A. Place gauze in the patient’s mouth.
B. Place the patient in Trendelenburg position.
C. Protect the patient’s head and lower her to the floor.
D. Observe the patient in the chair for at least 45 minutes after the seizure.
E. Call for help.

81. After a phlebotomy draw, the phlebotomist realizes that the tourniquet was on for an
extensive length of time. Which of the following is the most likely resulting complication?
A. hematoma
B. excessive bleeding
C. hemolysis
D. hemoconcentration
NCCT Phlebotomy Review PG34
82. While performing a venipuncture in the antecubital space, the patient verbalizes an extreme
amount of pain and the blood in the collection tube is noted to be bright red. Which of the
following sites is MOST likely to have been inadvertently punctured?
A. Ulnar artery
B. Basilic vein
C. Cephalic vein
D. Brachial artery

83. The phlebotomist arrives at an inpatient room to collect a blood sample. There is an
isolation cart located outside the room. The phlebotomist washes her hands and puts on the
following personal protective equipment: gown, gloves, and mask. Which of the following types
of precautions are being used for this patient?
A. Contact Droplet Airborne
B. Contact Airborne
C. Standard Airborne
D. Contact Droplet

84. For which of the following tubes must the phlebotomist prep the site with povidone iodine or
chlorhexidine?
A. A yellow tube with ACD
B. A yellow tube with SPS
C. A lavender tube with EDTA
D. A light blue tube with sodium citrate

85. The phlebotomist received an order for a neonatal screen for PKU. Which of the following
equipment will be necessary? (Select the two (2) correct answers.)
A. filter paper collection card
B. 23g butterfly needle and adapter
C. lancet (depth 2.0mm)
D. 23g vacutainer needle and syringe
E. microtainer (green top)

86. Which of the following organizations makes on-site visits to inspect phlebotomy
laboratories?
A. FDA
B. DEA
C. CLIA
D. ASCLS
NCCT Phlebotomy Review PG35
87. Which of the following items should be used to prepare the site for a blood alcohol
collection? (Select the two (2) correct answers.)
A. hydrogen peroxide
B. Purell wipes
C. Chloraprep
D. iodine swabs
E. green surgical soap

88. A phlebotomist is tasked with drawing routine blood work on a patient, but upon entering the
room the phlebotomist notices a sign that states “No Bloodwork From Right Arm”. After reading
the sign the phlebotomist notices an IV in the patient's left arm. Under normal circumstances
which of the following is the best site for the phlebotomist to choose?
A. proximal to the IV site
B. the back of the right hand
C. the foot
D. distal to the IV site

89. Which of the following is the correct method for transferring blood from a syringe into
evacuated tubes?
A. Remove the hypodermic needle from the syringe after activating the safety device and
transfer the blood into capped evacuated tubes using a syringe transfer device.
B. Uncap the evacuated tubes, place them into a tube rack and transfer blood from the syringe
using a syringe transfer device.
C. Remove the hypodermic needle from the syringe and transfer the blood into capped
evacuated tubes by pushing on the syringe plunger.
D. Uncap the evacuated tubes, place them into a tube rack and transfer the blood from the
syringe through the hypodermic needle.

90. Which of the following information will a phlebotomist need to add onto the computer
generated label?
A. patient’s date of birth
B. patient’s first and last name
C. initials of the person collecting the sample
D. identification number

91. The phlebotomist obtains 70% isopropyl wipe, a 25g butterfly needle, and two evacuated
tubes to collect CBC and ethanol levels. Which of the following pre-analytical sources of error
are likely results of this collection? (Select the two (2) correct answers.)
A. microclots
B. infection
C. specimen contamination
D. hemolysis
E. hemoconcentration
NCCT Phlebotomy Review PG36
92. Which of the following is a site preparation solution required for collecting blood cultures and
blood alcohol levels?
A. Isopropyl alcohol
B. Hydrogen peroxide
C. Soap and water
D. Povidone-iodine

93. Which of the following would be considered a hospital acquired infection?


A. A 72-year-old patient in the ER with flu-like symptoms.
B. An 8-year-old patient admitted with chicken pox.
C. A patient who contracts a MRSA infection from a venipuncture.
D. A healthcare worker that develops Hepatitis C.

94. Negligence by a healthcare professional is considered which of the following?


A. Battery
B. Assault
C. Malpractice
D. Non-maleficence

95. Which of the following situations requires corrective action before test results can be given
to the doctor?
A. Both controls are between 1 SD and 2 SD on the high side and are drifting up each day.
B. Normal glucose is in control at 2 SD on the low side and abnormal glucose at 2 SD on the
high side.
C. Normal glucose is at 3 SD on the high side and abnormal glucose is 2 SD on the low side.
D. Both controls are between 1 SD and 2 SD on the low side.

96. To anchor the vein a phlebotomist should use the


A. index finger above and thumb below the site.
B. thumb 1-2 inches above the site.
C. index finger and pull the skin taut.
D. thumb 1-2 inches below the site.

97. During an evacuated tube system draw, the phlebotomy technician properly positions the
needle in the vein and inserts the blood tube into the tube holder to collect a blood sample, but
there is no blood return. Which of the following should the phlebotomy technician do next to
obtain the specimen?
A. Replace the initial tube with another tube to check vacuum.
B. Use a different blood collection system to collect the blood.
C. Adjust the needle to gain blood flow.
D. Remove the needle and start the procedure again.
NCCT Phlebotomy Review PG37
98. The right side of a patient’s body is severely burned, including the right arm and hand. The
patient has an IV in the antecubital area of the left arm. The physician orders labs that require
venous blood. Which of the following should the phlebotomist do?
A. Turn off the IV for 2 minutes and collect the specimen distal to the IV in the antecubital area
of the left arm.
B. Collect distal to the IV in the antecubital area of the left arm after the nurse has turned off the
IV for 2 minutes.
C. Collect distal to the IV in the antecubital area of the left arm after the nurse has disconnected
the IV.
D. Disconnect the IV and collect the specimen distal to the IV in the antecubital area of the left
arm.

99. The phlebotomist was struck by a contaminated needle while changing out an overflowing
sharps container. The phlebotomist should immediately
A. wash the exposed area with soap and water.
B. soak the puncture site in BetadineTM.
C. milk the puncture site and clean with alcohol.
D. report the incident to a supervisor.

100. What is the NEXT step when a STAT test has been ordered for the patient, but the patient
is not in their room?
A. Consult the nurse
B. Postpone the collection
C. Locate the patient
D. Reschedule the test

101. Which of the following statements by a new phlebotomist indicates the need for further
education on standard precautions?
A. "I will wear a gown, gloves, and mask for inpatient collections.“
B. “Antimicrobial wipes may be used for cleaning a collection area at the end of a shift.”
C. "Sharps containers should be replaced when 2/3 full."
D. "I will perform hand hygiene after removing gloves."

102. Using the guidelines for any method of venipuncture, which of the following actions should
the phlebotomist perform FIRST?
A. Position the patient’s arm and cleanse the site with an antiseptic wipe.
B. Explain the procedure to the patient.
C. Review the requirements for collecting and handling the blood specimen.
D. Apply the tourniquet and thoroughly palpate the selected vein.
NCCT Phlebotomy Review PG38
103. A phlebotomist is working at a cardiology office where the majority of the patients are on
either aspirin or anticoagulant therapy. Upon retracting the needle from the patient’s arm the
patient begins to bleed profusely. Which of the following should the phlebotomist do next?
A. Have the patient hold direct pressure for up to 5 minutes.
B. Apply a pressure bandage.
C. Leave the patient and immediately notify the doctor.
D. Hold direct pressure for up to 5 minutes.

104. The phlebotomist is given an order for a blood draw. How high above the venipuncture site
should the phlebotomist wrap the tourniquet around the arm?
A. 1-2 cm
B. 1-2 inches
C. 3-4 cm
D. 3-4 inches

105. How often should QC be run on a blood glucose meter?


A. per manufacturer's instructions
B. every 5 days
C. every 10 days
D. every 30 days

106. A patient brings in written lab test results, including a mix of POC and higher complexity
tests, and asks the phlebotomist for help with interpretation. Which of the following should the
phlebotomist do?
A. Explain the interpretation behind the POC care tests, but refer the patient to the physician for
the higher complexity tests.
B. Explain the interpretation behind the POC tests, but let the patient read the directions with the
higher level tests.
C. Hand the patient pre-printed test explanations downloaded from the CDC web site.
D. Refer the patient to the physician for interpretation of all results.

107. While performing a venipuncture, the phlebotomist removes the filled tubes, but forgets to
remove the tourniquet prior to needle withdrawal. Which of the following is the MOST likely
result?
A. Syncope
B. Hemolyzed specimen
C. Hemoconcentration specimen
D. Hematoma

108. A lactic acid specimen should be delivered to the lab as


A. routine and on ice slurry.
B. STAT and on ice slurry.
C. STAT and protected from light.
D. routine and protected from light.
NCCT Phlebotomy Review PG39
109. When performing a venipuncture, the phlebotomist is stuck with a bloody needle upon
withdrawal from the patient. What is the FIRST thing the phlebotomist should do?
A. Report to a supervisor.
B. Seek immediate medical attention.
C. Document the incident.
D. Wash the exposed area with soap and water.

110. Which of the following is outside the scope of practice for a phlebotomist?
A. Prioritizing collections based on order requests.
B. Performing CLIA high complexity tests.
C. Recording quality control results on a log sheet.
D. Verifying equipment function on point of care testing using a control.

111. An older adult patient arrives to have blood drawn. While assessing the arms for
venipuncture, the phlebotomist notices scar tissue in the antecubital areas and is unable to
palpate a vein. Which of the following is an appropriate alternative course of action?
A. Perform a capillary puncture on the lateral thumb.
B. Use a larger gauge needle to penetrate through the scar tissue.
C. Draw blood from the hand using an evacuated tube system.
D. Draw blood from the hand using a winged infusion set.

112. Evacuated tubes must be filled to the level indicated. This will result in the correct ratio of
A. air to liquid.
B. additive to blood.
C. blood to air.
D. liquid to additive.

113. Which of the following statements represents correct patient identification for a
phlebotomist in an outpatient setting?
A. "May I have your name please?"
B. "Ms. Smith, I need to see your driver's license."
C. “Is your name Evelyn Smith?”
D. "Is your name Ms. Smith?"

114. Which of the following applies to every United States clinical laboratory that tests human
specimens?
A. College of American Pathologists (CAP)
B. Commission on Office Laboratory Accreditation (COLA)
C. Joint Commission on the Accreditation of HealthCare Organizations (JCAHO)
D. Clinical Laboratory Improvement Amendments (CLIA)
NCCT Phlebotomy Review PG40
115. Which of the following should the phlebotomist do when collecting blood from a fingertip
using a blade-type device?
A. Milk the finger during the blood collection.
B. Ensure all alcohol is wiped off the finger before the puncture is made.
C. Turn the patient’s hand palm side up.
D. Make the puncture perpendicular to the fingerprint whorls.

116. Capillary blood specimens are MOST likely to be successfully collected from patients that
A. have peripheral edema.
B. have poor peripheral circulation.
C. are extremely obese.
D. are dehydrated.

117. The phlebotomist has a requisition for chemistry and coagulation testing. Which of the
following is the correct tube selection and order of draw?
A. Red, royal blue
B. SST, royal blue
C. Light blue, SST
D. Red, light blue

118. During a phlebotomy collection, the needle came out of the patient's arm and the
phlebotomist’s gloves were spattered with blood. Which of the following statements correctly
describes the disposal of the waste?
A. First dispose of the sharps in puncture-proof biohazard sharps container, second dispose of
the contaminated gloves in the trash can.
B. First dispose of the sharps in puncture-proof biohazard sharps container, second dispose of
the contaminated gloves in a biohazard bag.
C. First dispose of the contaminated gloves in the trash can; second dispose of the sharps in a
puncture-proof biohazard sharps container.
D. First dispose of the contaminated gloves in a biohazard bag; second dispose of the sharps in
a puncture-proof biohazard sharps container.

119. A patient presents to have a diagnostic HIV test drawn. Which of the following forms of
consent must the phlebotomist obtain prior to venipuncture?
A. Verbal
B. Implied
C. Expressed
D. Informed
NCCT Phlebotomy Review PG41
120. The phlebotomist is instructed to perform a capillary stick for newborn screening. Which of
the following collection devices should the phlebotomist use?
A. Butterfly needle and syringe
B. Finger stick lancet
C. Heel stick lancet
D. Evacuated tube

121. While performing a venipuncture, immediately before removing the needle, the
phlebotomist should
A. gently invert the evacuated tubes.
B. activate the needle safety mechanism.
C. apply a gauze pressure dressing.
D. release the tourniquet.

122. When performing venipuncture on a patient with a left mastectomy, which of the following
sites should the phlebotomist use?
A. Right saphenous vein
B. Right antecubital fossa
C. Left basilic vein
D. Back of left hand

123. When disposing of hazardous materials, the phlebotomist must adhere to the guidelines
and standards set forth by
A. CMS.
B. CLIA.
C. FDA.
D. OSHA.

124. According to the CLSI recommended order of draw for capillary puncture, which of the
following microcollection tubes must be collected first?
A. sodium heparin
B. potassium EDTA
C. potassium oxalate
D. no additive

125. Which of the following should a phlebotomist do when testing glucometers for quality
assurance?
A. record patient results
B. compare policy and procedures
C. train patients
D. maintain records
NCCT Phlebotomy Review PG42
126. On the initial attempt the phlebotomist went through the vein, but was able to pull back on
the needle and successfully collect the specimen. Which of the following should the
phlebotomist do NEXT?
A. Have the patient hold pressure on the site for at least 10 minutes and notify the nurse.
B. Apply an ice pack to the site to minimize bleeding and prevent hematoma.
C. Apply a pressure dressing and instruct the patient to leave it on for 24 hours.
D. Explain to the patient that a bruise may be likely, but it should be gone within a few days.

127. The physician calls the lab questioning the electrolyte results of a patient the phlebotomist
has just drawn. One of the results went from a normal level to critically high. Which of the
following collection errors is MOST likely to have occurred?
A. The specimen was hemolyzed.
B. The specimen was collected in lithium heparin.
C. The specimen was clotted.
D. The specimen was drawn below the IV site.

128. Cryoglobulin specimens must be transported to the laboratory


A. protected from light.
B. immersed in ice.
C. at room temperature.
D. with a 37 degree Celsius heat block.

129. When leaving a droplet isolation room, which of the following is the correct order for
removing PPE?
A. mask, gown, gloves
B. mask, gloves, gown
C. gown, mask, gloves
D. gloves, gown, mask

130. The phlebotomist is asked to draw blood from a patient with an IV in the right arm whose
left arm is inaccessible. The phlebotomist may draw without a physician’s approval from the
A. IV line.
B. right arm proximal to the IV site.
C. foot.
D. right arm distal to the IV site.

131. The phlebotomist has collected an ammonia level. Which of the following should be done
regarding transport of the specimen?
A. Wrap the specimen in a heel warmer to slow the metabolic process.
B. Wrap the specimen in foil to protect from light.
C. Immerse the specimen in an ice and water slurry.
D. Maintain the specimen at room temperature.
NCCT Phlebotomy Review PG43
132. Before drawing a patient, the phlebotomist notices a droplet of dried blood on the arm of
the phlebotomy chair. Which of the following should the phlebotomist do?
A. Use an alcohol pad to wipe the dried droplet of blood, then dry with a paper towel.
B. Moisten the droplet with water, clean with antimicrobial soap, then dry with a paper towel.
C. Use a gloved hand to scrape up the blood, clean with disinfectant, then absorb with a paper
towel.
D. Use disinfectant to moisten the droplet, absorb with a paper towel, then clean with
disinfectant.

133. An infant is brought to the clinic for a CBC. After performing a heel stick, not enough blood
is collected to perform the test. Which of the following actions should the phlebotomist take?
A. Perform a fingerstick.
B. Apply a warm compress.
C. Add one drop of diluent to the bottom of the microtainer.
D. Use a deeper lancet.

134. Which of the following is the proper draw site for an infant less than one year of age?
A. lateral surface of heel
B. palmar surface of hand
C. 4th distal phalanx
D. plantar surface of heel

135. A phlebotomist spills a reactive agent in the lab. Which of the following should be
referenced for clean up?
A. product inserts
B. site supervisor
C. Material Safety Data Sheet
D. OSHA guidelines

136. A patient arrives with a family member for a blood draw. The phlebotomist notices the
patient speaking in another language, but the family member was responding to her
intermittently in English. The phlebotomist should
A. use non-verbal cues to accomplish the task.
B. call for a professional interpreter.
C. inform the supervisor.
D. allow the family member to serve as an interpreter.

137. Which of the following is the minimum PPE requirement when drawing lab work on a
patient with suspected HIV?
A. Gown, gloves, and mask
B. Gloves and mask only
C. Gloves only
D. Mask only
NCCT Phlebotomy Review PG44

138. A phlebotomist working in a hospital is collecting blood from a patient with very small,
difficult veins. Three tubes of blood need to be collected. The blue and green tubes fill easily
and completely, but the lavender tube fills incompletely. Changing the lavender tube using the
same draw also results in an incomplete specimen. Which of the following should the
phlebotomist do NEXT?
A. Re-position the needle and recollect the lavender tube.
B. Submit the samples as drawn.
C. Re-draw the patient using a different site and obtain all three tubes.
D. Resubmit the draw order for the next round of draws.

139. A 19-year-old patient offers her arm to the phlebotomist when she comes in with a
venipuncture tray. This is an example of
A. implied consent.
B. informed consent.
C. parental consent.
D. verbal consent.

140. The phlebotomist has an order to collect a complete blood count with differential, partial
thromboplastin time, basic metabolic panel, and blood cultures. Place the collection tubes in the
correct order of draw.
Blood culture bottles
Light blue
Lavender
Red

141. Which of the following standards are being violated when a phlebotomist fails to use proper
handwashing technique?
A. NAACLS
B. HIPAA
C. OSHA
D. CMS

142. Capillary blood specimen collection is contraindicated in patients that


A. are extremely obese.
B. have very difficult veins.
C. are severely burned over the torso.
D. have edema of the hands and feet.
NCCT Phlebotomy Review PG45

143. A phlebotomist was collecting a CBC and BMP on a patient with a prominent vein, when
the serum tube did not fill. After redirecting and trying another serum tube which did not fill, the
phlebotomist went on to collect a PST tube and an EDTA tube and filled them completely.
Which of the following is the MOST likely reason the serum tubes did not fill?
A. The wrong tube type was collected.
B. An expired tube was collected.
C. The tubes were collected in the incorrect order of draw.
D. The seal on the needle was broken.

144. What is the MOST common site to draw an arterial blood collection?
A. Brachial artery
B. Hepatic artery
C. Radial artery
D. Peroneal artery

145. Place the options below in the correct sequence for applying PPE. (Click and drag the
options in the left column to the correct position in the right column.)
Gloves
Gown
Mask
Face shield

146. A patient notices this symbol on a container in the room and asks what it
means. The phlebotomist should tell the patient the symbol means
A. isolation precautions required.
B. radiation in use.
C. emergency eye wash station.
D. biohazard material present.

147. The phlebotomist is collecting a capillary sample from an infant. Following the collection,
the phlebotomist holds firm pressure on the site until bleeding has stopped. How should the
phlebotomist proceed?
A. Bandage with a pressure dressing.
B. Tape a sterile gauze pad over the site.
C. Do not bandage.
D. Put a cartoon character bandage on the site.
NCCT Phlebotomy Review PG46

148. When preparing to do a urine drug test, which of the following should the phlebotomist ask
the patient?
A. “What time was your last meal?”
B. “Are you taking any medications?”
C. “Do you have any diet restrictions?”
D. “Have you been standing for 30 minutes?”

149. The phlebotomist is educating the patient on fasting diet requirements prior to a glucose
tolerance test. Which of the following is the minimum time frame the patient must remain fasting
before the start of the test?
A. 13-24 hours
B. 3-4 hours
C. 1-2 hours
D. 8-12 hours

150. When inserting a needle during venipuncture, the bevel should be in which of the following
positions in relation to the vein?
A. Left
B. Up
C. Down
D. Right
NCCT Phlebotomy Review PG47

Study Guide - Collections 92Q’s


1. Which of the following specimens is likely to be considered STAT?
A. therapeutic drug levels
B. Lactic acid
C. Electrolytes
D. Blood smears

2. A phlebotomist is assigned to manage an outpatient draw station. Prioritize the following


patients in the correct order:
2 Hour postprandial glucose level due in 40 minutes
Type and screen on a pre-op patient due for surgery that day
Weekly PT/INR
Stat troponin level on a patient sent over from a doctors office

3. A phlebotomist performs a blood draw testing for CBC, chemistry, and bilirubin. After the
draw, the phlebotomist wraps the bilirubin tube in aluminum foil. Which of the following was the
reason for the wrapping of the bilirubin tube?
A. To cool down the specimen.
B. To maintain the specimen at 37 degrees Celsius.
C. To protect specimens from light.
D. To prevent contamination of the specimen.

4. Which of the following correctly matches the test with the appropriate special handling
instructions?
A. vitamin A specimen kept warm using a heating pack
B. cryoglobulin level protected from light with aluminum foil
C. ammonia level kept warm using a heating pack
D. lactic acid chilled in slurry of ice and water

5. The phlebotomist draws blood from a patient in an isolation room. The specimen should be
properly taken to the lab in a
A. red biohazard bag
B. yellow bag
C. biohazard transport bag
D. brown paper bag
NCCT Phlebotomy Review PG48
6. Which of the following samples is still acceptable when kept at room temperature for 8 hours
after collection?
A. a urine sample
B. a stool culture
C. a glucose test in a gray top tube
D. a B12 test in a red top tube

7. Which of the following blood tests require special handling? (Select the three (3) correct
answers.)
A. CBC
B. ammonia
C. cold agglutinin
D. blood culture
E. bilirubin

8. A glucose specimen collected in a gel barrier tube from a hyperglycemic patient was left in a
pneumatic tube system for four hours before being centrifuged. The test results were unusually
low. Which of the following specimen handling errors is most likely to have caused the
inaccurate results?
A. The specimen was not protected from light and cells were not separated from serum or
plasma within the time limit.
B. The specimen was not protected from light and was not placed on ice.
C. The specimen was transported in a pneumatic tube system and was not placed on ice.
D. The specimen was transported in a pneumatic tube system and cells were not separated
from serum or plasma within the time limit.

9. When processing a specimen in a centrifuge the phlebotomist should ensure that all the
A. samples are centrifuged at 3000 rpm.
B. samples are balanced in the centrifuge.
C. samples are spun for at least 20-30 minutes.
D. tubes are allowed to clot for 10 minutes before spinning.

10. Which of the following tubes needs time to clot before being centrifuged?
A. red
B. gray
C. green
D. light blue

11. Which of the following tubes is most likely to be centrifuged?


A. tiger top SST
B. yellow ACD
C. royal blue EDTA
D. yellow SPS
NCCT Phlebotomy Review PG49
12. Which of the following should the phlebotomist do if urine testing will be delayed more than
an hour?
A. Place the urine on ice, then in the refrigerator.
B. Protect the specimen from light.
C. Place the urine specimen in the refrigerator.
D. Store the urine at room temperature.

13. A phlebotomist is asked to process a bilirubin level collected in a red top tube. The
phlebotomist should centrifuge the specimen
A. at least 20 minutes after collection and aliquot into an amber tube.
B. at least 20 minutes after collection and aliquot into a plain tube.
C. 60 minutes after collection and aliquot into an amber tube.
D. 60 minutes after collection and aliquot into a plain tube.

14. The phlebotomist collects electrolytes and ammonia specimens from the same patient in
lithium heparin tubes, and places them in ice slurry while collecting other specimens before
transporting them to the lab 15 minutes later. The results for electrolytes were incorrect, most
likely because the specimen was
A. collected in the wrong tube.
B. not processed in a timely manner.
C. not transported at the correct temperature.
D. not protected from light.

15. Which of the following urine samples should be used for a cortisol level?
A. timed
B. 24 hour
C. random
D. clean catch

16. After a tiger top tube was centrifuged the phlebotomist noticed that the serum was milky
white. Which of the following conditions does this indicate?
A. diabetes
B. anemia
C. icterus
D. lipemia

17. A phlebotomist has just drawn a chemistry tube on an inpatient at the hospital. The tube has
been spun down and opened so that an aliquot could be taken from the sample. A request for a
blood alcohol level has been ordered on the same patient. Which of the following should the
phlebotomist do?
A. Add the test to the aliquot.
B. Draw a new chemistry tube on the patient.
C. Call for permission to add the test to the aliquot.
D. Allow the patient sufficient time before drawing a new tube.
NCCT Phlebotomy Review PG50
18. A phlebotomist in central processing discovers an unspun glucose specimen in a gray top
tube kept at room temperature for four hours after it was collected. The phlebotomist should
A. transfer the specimen to the chemistry department for testing.
B. transfer the specimen to the centrifuge to be spun down.
C. reject the specimen due to the delay in processing.
D. rejected the specimen because the temperature requirement was not met.

19. A specimen processor receives a lipemic glucose specimen, a clotted HCG, a moderately
hemolyzed electrolyte panel specimen, and an icteric bilirubin specimen. Which of the following
is likely to be rejected for analysis?
A. glucose and bilirubin
B. glucose and electrolytes
C. HCG and bilirubin
D. HCG and electrolytes

20. Which of the following laboratory departments should a Pap smear be delivered to?
A. Hematology
B. Immunology
C. Microbiology
D. Cytology

21. Which clinical laboratory department do ANA, HIV, and HCG specimens go to?
A. Microbiology
B. Biochemistry
C. Serology/Immunology
D. Hematology

22. Which department within the lab should a BMP be delivered to?
A. Serology Department
B. Chemistry Department
C. Blood Bank Department
D. Coagulation Department

23. Which of the following laboratory departments should a rapid plasma reagin (RPR) be
delivered to?
A. Hematology
B. Microbiology
C. Immunology
D. Histology
NCCT Phlebotomy Review PG51
24. A phlebotomist collects citrate, EDTA, and oxalate tubes in that order. Which of the
following is the correct lab department for each tube?
A. Citrate to coagulation, EDTA to hematology, and oxalate to chemistry.
B. Citrate to hematology, EDTA to hematology, and oxalate to immunology.
C. Citrate to coagulation, EDTA to immunology, and oxalate to chemistry.
D. Citrate to hematology, EDTA to immunology, and oxalate to chemistry.

25. Which of the following is the appropriate antiseptic to use in order to achieve a high degree
of antisepsis prior to blood culture collection on a 6-month-old infant?
A. benzalkonium chloride
B. povidone-iodine (PVP)
C. betadine
D. chlorhexidine gluconate

26. The phlebotomist has orders to draw blood cultures on a patient with fragile veins and
decides to use a butterfly system. Which of the following should be drawn first?
A. anaerobic
B. aerobic
C. pediatric
D. mycobacterium

27. Which of the following is the reason a phlebotomist should avoid milking a finger during a
capillary puncture?
A. The specimen may be contaminated with interstitial fluid.
B. The specimen may be contaminated with staphylococcus aureus.
C. The specimen may be contaminated with isopropyl and cause hemolysis.
D. The collection site may be contaminated by the phlebotomist's gloves.

28. A three-month-old patient was sent to the lab for an H&H. How should the phlebotomist
collect the sample?
A. venipuncture, antecubital fossa
B. dermal puncture, great toe
C. dermal puncture, middle finger
D. dermal puncture, heel stick

29. Which of the following tests can be collected by capillary puncture? (Select the two (2)
correct answers.)
A. blood cultures
B. blood gas
C. bilirubin
D. erythrocyte sedimentation rate
E. PTT
NCCT Phlebotomy Review PG52
30. If alcohol is used to decontaminate the capillary puncture site, which of the following should
the phlebotomist do?
A. Use two alcohol pads for cleaning.
B. Dilute the strength of the alcohol.
C. Wipe away the 2nd drop after the puncture.
D. Allow the site to air dry before the puncture.

31. Which of the following is the correct order for filling micro collection tubes when multiple
tests are ordered?
A. EDTA, non-additive tubes, other additive tubes
B. non-additive tubes, EDTA, other additive tubes
C. EDTA, other additive tubes, non-additive tubes
D. non-additive tubes, other additive tubes, EDTA

32. A phlebotomist is tasked with drawing a STAT CBC and a CMP. After palpation, no veins
can be felt and there are no visible veins on the back of the hand. The phlebotomist decides to
perform a capillary stick. Which of the following microtainers should the phlebotomist choose?
A. 1 polymer gel and 1 without an additive.
B. 1 EDTA and 1 heparin.
C. 1 polymer gel and 1 EDTA.
D. 1 heparin and 1 without an additive.

33. The doctor has ordered a fecal occult blood specimen to be collected by the patient. Which
of the following preparatory instructions should the phlebotomist give the patient?
A. Consume 8 ounces of fiber before collection.
B. Avoid taking vitamin C prior to collection.
C. Stop taking prescription medication 12 hours prior to collection.
D. Fast for eight hours before collection.

34. While providing patient education on 24-hour urine collection for creatinine clearance, it is
imperative to tell the patient
A. to keep the specimen refrigerated at all times.
B. to maintain particular dietary restrictions.
C. start collection 96 hours after any procedure using contrast dye.
D. to avoid laxatives during the collection process.

35. Which of the following are required for a sputum collection?


A. sterile cup, 24 hour collection
B. plastic transport cup, immediate delivery to lab
C. sterile cup, immediate delivery to lab
D. plastic transport cup, 24-hour collection
NCCT Phlebotomy Review PG53
36. A patient arrives at an outpatient draw station for a glucose tolerance test collection. The
phlebotomist has verified that the patient has been fasting for 10 hours. Place the steps below
in the correct order for performing the test.
A. Instruct the patient to drink the predetermined dose of the glucose beverage.
B. Collect the initial blood specimens for glucose and insulin levels.
C. Collect blood specimens at physician prescribed intervals.
D. Start the timing for the glucose tolerance test.

37. After a course of antibiotics for a UTI a patient returns for a follow up urinalysis. The order
calls for a midstream clean catch specimen. In order to prevent contamination, the phlebotomist
should provide the patient with which of the following antiseptics?
A. iodine wipes
B. castile soap wipes
C. betadine wipes
D. alcohol wipes

38. A patient who received a vasectomy 90 days ago reports to the lab for a semen analysis.
The patient will be collecting the sample at home. To avoid sample rejection the phlebotomist
should instruct the patient to keep the specimen
A. at body temperature and return it to the lab within 30 minutes.
B. at room temperature and return it to the lab within 30 minutes.
C. in a zip locked bag over ice and return it to the lab within 30 minutes.
D. in a zip lock bag over ice and return it to the lab within 8 hours.

39. Which of the following should the phlebotomist tell the patient when giving instructions to a
patient for sputum collection? (Select the two (2) correct answers.)
A. Collect a first morning specimen.
B. Collect the specimen 1 hour after eating a meal.
C. Gargle with mouthwash before collection.
D. Consume only clear liquids prior to collection.
E. Rinse the mouth with sterile water prior to collection.

40. A urine drug screen has been ordered for the patient and instructions for how to avoid it
have been provided. When the phlebotomist is given the specimen, she must immediately
A. prepare the specimen for transport.
B. read the temperature strip.
C. ask the patient what medications he is taking.
D. wash her hands.
NCCT Phlebotomy Review PG54
41. The physician has ordered a sputum collection. Which of the following patient instructions
must be given before collection of the sputum?
A. “Breathe deeply in and out.”
B. “Rinse the mouth with warm water.”
C. “Cough several times.”
D. “Brush your teeth and use mouthwash.”

42. A phlebotomist working in an outpatient setting has just received a set of guaiac smears
from a patient that must be processed. How should the phlebotomist process the specimen?
(Select the two (2) correct answers.)
A. Develop with 70% isopropyl alcohol.
B. Keep at room temperature.
C. Develop with hydrogen peroxide.
D. Keep chilled.
E. Develop with normal saline.

43. Which of the following sites is contraindicated for a patient with diabetes?
A. antecubital area
B. foot/ankle
C. hand
D. lower forearm

44. Which of the following types of medication work to inhibit blood from clotting?
A. antihypertensives
B. anticoagulants
C. antiglycolytic
D. antihistamines

45. A patient visits the lab for a routine venipuncture collection and informs the phlebotomist that
she recently had a mastectomy of the right breast. Which of the following is the appropriate
draw site?
A. right arm
B. left arm
C. either arm
D. either hand

46. During a venipuncture the patient has excessive bleeding and then states she forgot to
mention her anticoagulant (Coumadin) therapy. Which of the following should the phlebotomist
do immediately after removing the needle?
A. Immediately apply a pressure bandage.
B. Ask the patient to hold pressure on the site and then look for a physician.
C. Hold pressure on the site for a minimum of five full minutes until the bleeding stops.
D. Hold pressure on the site until the bleeding begins to stop and apply a bandage.
NCCT Phlebotomy Review PG55
47. A phlebotomist is to draw blood from a psychiatric patient. The patient does not understand
what is being done. Which of the following is the appropriate action the phlebotomist should
take to ensure the safety of all?
A. Perform the blood draw in the presence of the physician.
B. Explain the procedure to the patient thoroughly.
C. Confirm the patient's identity and perform the draw.
D. Take patient restraints into the room.

48. When performing a blood draw on a 75-year-old patient with thin, papery skin, the
phlebotomist should
A. apply the tourniquet directly to the skin above the site
B. wrap the arm in gauze and apply the tourniquet
C. do not apply a tourniquet
D. use a cloth tourniquet

49. The phlebotomist has orders to draw blood on a patient with edema in the left arm and
burns on the upper right arm. Which of the following is the preferred draw site?
A. foot vein
B. antecubital vein
C. hand vein
D. femoral vein

50. Which of the following should the phlebotomist do with a patient known to have a fainting
problem?
A. Inform the physician that the patient cannot be drawn.
B. Give the patient orange juice.
C. Position the patient supine.
D. Ask a coworker to assist.

51. Which of the following is a phlebotomist required to do when preparing to aliquot a


specimen? (Select the three (3) correct answers.)
A. Match the labels on the tubes.
B. Invert the tubes 8 to 10 times before centrifuging.
C. Prepare under a bench top splash shield.
D. Pour the specimen directly into the aliquot container.
E. Use a disposable transfer pipette.

52. Which of the following statements made by a phlebotomist indicates understanding of


proper specimen processing? (Select the two (2) correct answers.)
A. “I must pre-label aliquot tubes one patient at a time.”
B. “I must centrifuge a gray top tube for whole blood testing.”
C. “I must label aliquot tubes one patient at a time immediately after aliquoting.”
D. “I must aliquot a K level into an amber tube.”
E. “I must remove stoppers from tubes using a face shield.”
NCCT Phlebotomy Review PG56
53. A hospital phlebotomist is conducting routine glucose monitoring for a patient. Why should
the first drop of blood be wiped away prior to applying it to the test strip?
A. The 70% isopropyl can contaminate the blood sample and give elevated results.
B. The patient may have bacteria on their finger, which can cause abnormal results.
C. The first drop is over oxygenated.
D. The first drop of blood may contain fluid from the tissues.

54. Which of the following is a coagulation test that can be performed with a POC machine or
instrument?
A. HCT
B. BUN
C. D-dimer
D. INR

55. The doctor has ordered an occult blood test and sent the patient to the lab to pick up the
testing materials. After instructing the patient how to collect the specimen, it is very important to
tell him to
A. follow dietary guidelines specifically for the test.
B. inform the doctor about any gastric ulcers he may have.
C. clean the area thoroughly.
D. stop taking medications during the collection period.

56. Which of the following is a POCT?


A. cortisol
B. FOB
C. iron
D. T4

57. The phlebotomist notices that the patient's INR is in the critical value range. Which of the
following should the phlebotomist contact regarding the critical result?
A. The doctor who ordered the test.
B. The doctor’s nurse.
C. The doctor's medical assistant.
D. The lab manager.

58. Which of the following is the correct pairing of tube and test for a chemistry specimen
collection?
A. gold and blood type
B. tiger top and lactic acid
C. green and AST
D. yellow and D-dimer
NCCT Phlebotomy Review PG57
59. When ordering a PT, PSA and CBC, which of the following tubes are needed to effectively
draw these labs? (Select the three (3) correct answers.)
A. light blue
B. gray
C. lavender
D. gold gel barrier
E. yellow ACD

60. The doctor has ordered a CBC, Cholesterol, and PT on a fasting patient. The phlebotomist
is in the process of collecting the last tube when the nurse walks in with an add-on for a glucose
test. Which of the following tubes should the phlebotomist use to collect the glucose test?
A. light green top
B. SST
C. lavender top
D. gray top

61. Which of the following tubes contains an anticoagulant that prevents platelets from clumping
and preserves the appearance of blood cells for microscopic prep?
A. light green
B. dark green
C. light blue
D. lavender

62. A phlebotomist is drawing a sample for glucose testing and realizes she is using a pink top
sample tube. Which of the following should the phlebotomist do?
A. Quarantine the sample.
B. Discontinue the draw and reschedule the patient.
C. Continue the draw using a black top.
D. Continue to draw using a gray top.

63. The phlebotomist has orders for an FBS, INR, and CBC Stat. Which of the following is the
correct tube selection and order of draw?
A. red, lavender, light blue
B. red, light blue, lavender
C. lavender, light blue, gray
D. light blue, lavender, gray

64. A pregnant woman presents with orders for an oral glucose gestational challenge test. After
the patient finishes the drink, how long does the phlebotomist need to wait before drawing the
specimen?
A. 180 min
B. 60 min
C. 90 min
D. 120 min
NCCT Phlebotomy Review PG58
65. When collecting blood for a neonatal screening, the phlebotomist should
A. use a capillary collection procedure.
B. use special patient identification banding.
C. maintain a warm temperature at the site.
D. use a syringe method.

66. A patient with Pseudomonas aeruginosa is receiving gentamicin antibiotic for therapy. In
order to appropriately monitor this drug for patient safety the phlebotomist must
A. stay with the patient the initial two hours to monitor status.
B. collect two trough samples.
C. collect the blood sample before the drug is administered.
D. label all tubes simultaneously before drug monitoring starts.

67. A patient is receiving therapeutic medication. Which of the following is the correct way to
draw trough levels?
A. 60 minutes prior to medication administration
B. 30 minutes after medication administration
C. 30 minutes prior to medication administration
D. 10 minutes after medication administration

68. A trough Dilantin (phenytoin) level is ordered to be collected at 12:30 PM. The phlebotomist
arrives at 12:20 PM to collect the specimen and notices the patient is receiving an infusion of
normal saline in the right arm and Dilantin in the left arm. Which of the following is the
appropriate course of action?
A. Collect the specimen distal to the normal saline infusion and label it as peak level.
B. Verify the time of collection with the patient’s nurse or physician.
C. Consult the patient’s chart to determine appropriate collection time.
D. Contact the laboratory supervisor to confirm peak or trough collection.

69. Which of the following is the best way to properly anchor the vein?
A. Use the thumb and index finger to stretch the skin.
B. Place the thumb 1 to 2 inches below the site.
C. Use two fingers and stretch the skin toward the forearm.
D. Hold skin with your thumb tightly and pull forward toward your forearm.

70. When anchoring a vein the phlebotomist should (Select the two (2) correct answers.)
A. place the thumb 1-2 inches below the venipuncture site.
B. apply direct downward pressure.
C. draw the skin away from the site.
D. place the pointer finger 1-2 inches below the venipuncture site.
E. place the thumb below and the index finger above the site.
NCCT Phlebotomy Review PG59
71. A new bariatric patient has come to the lab for CMP and CBC. Inspection of both arms did
not reveal any veins. The patient tells the phlebotomist that blood is usually drawn from her
hand. Which of the following should the phlebotomist use to draw blood from this patient’s
hand?
A. 21g needle at a 40 to 45 degree angle
B. 21g needle at a 10 to 15 degree angle
C. 23g needle at a 40 to 45 degree angle
D. 23g needle at a 10 to 15 degree angle

72. Which of the following specimens is most susceptible to contamination and should always
be drawn first?
A. hematocrit
B. serum cholesterol
C. complete blood count
D. blood culture

73. Place the tubes below in the correct order of draw.


EDTA
SST
SPS
Heparin

74. A phlebotomist has just gathered her supplies for an out-patient procedure and has selected
gray, red, light blue, and lavender tubes. Which of the following is the appropriate order of draw?
A. lactic acid, ESR, D-dimer, RPR
B. D-dimer, RPR, ESR, lactic acid
C. HIV, CTGC, D-dimer, RPR
D. D-dimer, RPR, CTGC, HIV

75. A requisition order requires a phlebotomist to draw a BMP, CBC, PTT, and lactic acid. Place
the options below in the correct order of draw.
CBC
Lactic acid
PTT
BMP

76. Which of the following is the correct order of draw when one or more tubes containing
anticoagulant are collected?
A. green, red, light blue
B. red, green, light blue
C. light blue, red, green
D. green, light blue, red
NCCT Phlebotomy Review PG60
77. Place the options below in the correct order of draw for micro containers.
Heparin
No additive
EDTA
Sodium fluoride

78. A phlebotomist must collect ESR, STAT electrolytes, D-Dimer and estrogen. Which of the
following correctly lists the appropriate tubes in the CLSI recommended order of draw?
A. red, light blue, green, lavender
B. light blue, red, green, lavender
C. light blue, red, lavender, green
D. red, light blue, lavender, green

79. The phlebotomist has to draw lavender, gold, light blue and gray top tubes for the following
tests. Place the tests listed below in the correct order of draw. (Click and drag the options in the
left column to their correct position in the right column.)
CBC
Cholesterol
Glucose
Prothrombin

80. A phlebotomist is tasked with drawing an ESR, CMP, PT/INR, and a lactic acid level. Which
of the following is the correct order of draw?
A. gold, lavender, pink, gray
B. gray, lavender, gold, light blue
C. pink, light blue, gray, gold
D. light blue, gold, lavender, gray

81. Which of the following is the CLSI recommended order of draw for a BMP, CBC & PTT?
A. EDTA, heparin, sodium citrate
B. heparin, EDTA, serum separator
C. serum separator, heparin, EDTA
D. sodium citrate, heparin, EDTA

82. Test orders are for hematology, serum chemistry, and coagulation studies. Which of the
following is the correct order of draw?
A. red, light blue, lavender
B. yellow, lavender, light blue
C. light blue, red, lavender
D. lavender, yellow, light blue
NCCT Phlebotomy Review PG61
83. The doctor ordered the following lab tests: CBC, calcium, glucose, ABO Group, Rh typing,
aPTT. Which of the following is the correct order of draw for these tests?
A. gold, lavender, pink, gray, light blue.
B. lavender, gray, gold, light blue, pink.
C. light blue, gold, lavender, pink, gray.
D. light blue, lavender, gold, pink, gray.

84. A phlebotomist is asked to collect a CBC, PT, electrolytes and BUN. The correct order of
draw for this collection is
A. light blue, green, lavender.
B. green, lavender, light blue.
C. lavender, light blue, green.
D. green, light blue, lavender.

85. A phlebotomist has completed a dermal puncture on a one-year-old patient who is


completing an annual physical. Children this age should not have a bandage applied because
A. they run a higher risk of contamination.
B. they run a risk of choking.
C. clotting occurs quickly for children this young.
D. they run a higher risk of petechiae.

86. A phlebotomist is preparing to draw an 85-year-old patient whose skin is thin and paper-like.
Which of the following should the phlebotomist use to secure the gauze?
A. paper tape
B. bandage
C. Micropore
D. Coban

87. A 76-year-old patient has fragile skin with little elasticity. The best choice to ensure his skin
doesn’t tear should be
A. silk tape.
B. paper tape.
C. surgical tape.
D. A Band-aid.

88. After drawing a patient with hemophilia the phlebotomist should


A. bandage the patient as normal.
B. apply an extra layer of gauze.
C. hold pressure for 5-10 minutes.
D. lower the patient's head.
NCCT Phlebotomy Review PG62
89. In order to prevent post-puncture hematoma formation at the antecubital fossa after a
successful venipuncture, the phlebotomist must apply firm and direct pressure with cotton
A. gauze and instruct the patient to bend the arm.
B. gauze and instruct the patient not to bend the arm.
C. balls and instruct the patient to bend the arm.
D. balls and instruct the patient not to bend the arm.

90. At what point should specimen labeling occur?


A. During assembly of the equipment.
B. Upon completion of the blood draw.
C. Before approaching the patient’s side.
D. After the patient is released.

91. The phlebotomist collects a small amount of blood from an elderly patient using a syringe.
There is not enough blood to fill all the collection tubes, so the phlebotomist places the blood in
the appropriate microcollection containers instead. Which of the following additional information
is required on the specimen label?
A. date and time of collection
B. phlebotomist’s initials
C. patient’s initials
D. source of specimen

92. The phlebotomist must label blood bank specimens with which of the following information?
(Select the three (3) correct answers.)
A. patient’s date of birth
B. patient’s room number
C. patient’s first and last name
D. date and time of collection
E. physician’s name
NCCT Phlebotomy Review PG63
Study Guide - Infection Control and Safety 53Q’s
1. A large open container of isopropyl alcohol is spilled on the floor in the clinic waiting area.
The phlebotomist should use which of the following resources for information about the
chemical and any necessary procedure?
A. Material Safety Data Sheet
B. Clinical Laboratory Improvement Amendments
C. Poison Control Center
D. Physician’s Desk Reference

2. Which of the following interventions is required by an employer, at no cost to the employee,


following a needlestick exposure?
A. Offer Hepatitis B vaccine if the needle was contaminated.
B. Place exposed personnel on antiviral medications.
C. Offer HIV testing to the employee’s spouse or significant other.
D. Provide a confidential medical evaluation

3. The phlebotomist encounters an unfamiliar chemical in the laboratory. Which of the following
should she reference for guidance in handling this chemical?
A. CDC
B. OSHA
C. NFPA
D. MSDS

4. The regulatory agencies for safety in health care settings include which of the following?
(Select the three (3) correct answers.)
A. OSHA
B. NFPA
C. CMS
D. CDC
E. CLIA

5. A patient was sneaking a cigarette in the bathroom of her room. The phlebotomist comes in
and notices a trash can on fire. Place the steps below in the correct order for dealing with the
fire. (Click and drag the options in the left column to their correct position in the right column.)
Extinguish the fire
Rescue the patient
Confine the fire
Activate the alarm
NCCT Phlebotomy Review PG64
6. The Needlestick Safety and Prevention Act, in conjunction with OSHA, mandates employers
to
A. provide immediate medical evaluation and scheduled surveillance.
B. protect access to medical records of the potential source individual.
C. provide educational inservices yearly on safe needle recapping and disposal.
D. have source individuals tested for blood borne pathogens.

7. Which of the following devices has the highest potential for a needlestick injury?
A. single sample needle
B. butterfly winged infusion set
C. lancets
D. syringes

8. Which of the following must be included when completing a phlebotomy related exposure
incident report?
A. the type of device used to collect the specimen
B. the type of tube used for the collection
C. the requisitioned test
D. the medical condition of the patient

9. If accidentally exposed to bloodborne pathogens, the phlebotomist’s first course of action


should be to
A. call the CDC to report the exposure.
B. report to an immediate supervisor.
C. wash the site with soap and water.
D. report directly to a healthcare provider for medical evaluation and treatment.

10. While performing a venipuncture the patient has a seizure and the needle accidentally sticks
the phlebotomist’s gloved hand. The phlebotomist should clean the site with
A. disinfectant for a minimum of 30 seconds.
B. antimicrobial soap and water for a minimum of 30 seconds.
C. 70% isopropyl alcohol for a minimum of 10 minutes.
D. sterile saline for a minimum of 10 minutes.

11. A phlebotomist fails to activate the needle guard after the draw, which results in an
accidental needle stick. Which of the following should immediately be used to cleanse the site?
A. disinfecting alcohol
B. soap and tap water
C. sterile saline
D. sanitizing agent
NCCT Phlebotomy Review PG65
12. The most important thing to do if the phlebotomist sustains a sharps injury is to
A. inform the patient.
B. look up her medical history.
C. file an incident report.
D. change gloves immediately.

13. After drawing a patient for an HIV, HBV and HCV antibody test, the phlebotomist
accidentally sticks his finger when discontinuing the venipuncture. Which of the following would
the phlebotomist do next?
A. Report the incident to the immediate supervisor.
B. Wash the site with a disinfectant for a minimum of 30 seconds.
C. Wash the site with soap and water for a minimum of 30 seconds.
D. Report directly to a licensed healthcare provider for treatment.

14. Which of the following actions should the phlebotomist take first after coming into direct
contact with a patient’s blood via needlestick injury?
A. Obtain the Hep B vaccine if not currently immunized.
B. Apply pressure at the site to stop the bleeding.
C. Flush the area with water and thoroughly wash it.
D. Inform the patient about the incident.

15. The syringe method has been used to draw an oncology patient. In the process of
transferring the blood from the syringe to the evacuated tube, the top of the tube pops off and
blood is splattered into the phlebotomist’s eyes. The first course of action is to flush the eyes
and then immediately
A. go to the ER.
B. report the incident to supervisor/manager.
C. have source blood tested for HBV and HIV.
D. have the phlebotomist’s blood tested for HBV and HIV.

16. While in the processing area, the phlebotomist accidentally breaks a vacuum tube filled with
blood on the counter, spilling it on the patient requisition form and the laboratory floor. Which of
the following actions should the phlebotomist take next?
A. Assess the counter to determine if it would be damaged by bleach and water.
B. Don gloves and gown.
C. Absorb the spill using paper towels.
D. Secure a biohazard container for disposal.

17. Place the options below in the correct order of response to a needle stick injury. (Click and
drag the options in the left column to their correct position in the right column.)
Wash with soap and water
Alert supervisor
Get consent from the patient
Report to employee health
NCCT Phlebotomy Review PG66
18. Which of the following chemicals is most commonly used to disinfect work surfaces and the
bottom of the phlebotomy tray?
A. benzalkonium chloride
B. chlorhexidine gluconate
C. isopropanol
D. bleach

19. When mixing a 10% bleach disinfectant solution following OSHA guidelines, the solution
ratio 1:10 should be mixed by adding
A. 70 ml water to 30 ml bleach.
B. 10 ml water to 90 ml bleach.
C. 9 ml water to 10 ml bleach.
D. 90 ml water to 10 ml bleach.

20. An unbalanced centrifuge causes a specimen collection tube to break inside of the device.
The phlebotomist should
A. sterilize the equipment.
B. discard the equipment.
C. disinfect the equipment.
D. sanitize the equipment.

21. CDC guidelines require a blood glucose meter to be disinfected


A. between patient use.
B. daily.
C. weekly.
D. hourly.

22. Which of the following should the phlebotomist use to disinfect equipment and facilities?
A. alcohol
B. antiseptic
C. iodine
D. bleach

23. Which of the following microorganisms is commonly associated with a hospital acquired
infection?
A. Staphylococcus epidermidis
B. Group A Streptococcus
C. Methicillin-resistant Staphylococcus aureus
D. Neisseria gonorrhoeae

24. Which of the following is the most common means of transmission of Hepatitis A?
A. sexual contact
B. contact with contaminated needles
C. contact with fecal contaminated food/water
NCCT Phlebotomy Review PG67
D. forceful coughing

25. A phlebotomist in a doctor’s office needs to perform a venipuncture on a patient who has
been diagnosed with Severe Acute Respiratory Syndrome. Which of the following is the
transmission based precaution the phlebotomist must take?
A. standard
B. droplet
C. airborne
D. contact

26. Which of the following measures is applicable when drawing blood on patients who have not
been identified as potentially infectious?
A. Contact Precautions
B. Standard Precautions
C. Droplet Precautions
D. Airborne Precautions

27. A phlebotomist has been asked to collect a specimen from a patient with bacterial
meningitis. Which of the following transmission-based precautions should be used?
A. contact
B. complete isolation
C. airborne
D. droplet

28. Which of the following diseases is a healthcare worker at the greatest risk of developing
following an exposure incident with blood from a needlestick injury?
A. HIV
B. HAV
C. HPV
D. HBV

29. A phlebotomist must collect a blood specimen from a patient with contact transmission
precautions. Which of the following is the best type of tourniquet the phlebotomist should use?
A. a disposable tourniquet
B. a blood pressure cuff
C. a pediatric tourniquet
D. a velcro tourniquet

30. The phlebotomist has a requisition to collect blood from a pediatric patient in a contact
isolation unit. The necessary PPE includes gloves and
A. face shield.
B. gown.
C. mask.
D. bonnet.
NCCT Phlebotomy Review PG68
31. The phlebotomist is entering a room marked with Contact Precautions. Which of the
following items must be worn? (Select the two (2) correct answers.)
A. gloves
B. mask
C. gown
D. N95 respirator
E. goggles

32. Which of the following actions by the phlebotomist caring for a patient in isolation is
contraindicated?
A. limiting the supplies in the room to only those necessary for the draw
B. bringing a phlebotomy tray into the room
C. leaving unused phlebotomy supplies in the room
D. removing PPE before completely exiting the room

33. A phlebotomist has received a requisition to collect a blood specimen on a patient infected
with influenza. The phlebotomist must be sure to follow
A. standard precautions and droplet precautions
B. droplet precautions and contact precautions
C. standard precautions and contact precautions
D. droplet precautions and airborne precautions

34. Which of the following illnesses fall under airborne transmission precautions? (Select the
three (3) correct answers.)
A. adenovirus
B. varicella
C. respiratory syncytial virus (RSV)
D. measles
E. MRSA

35. Which of the following equipment is used to draw blood on an infant?


A. 21 gauge needle
B. 2.0 mm lancet
C. 18 gauge needle
D. 2.5 mm lancet

36. After a venipuncture, a patient experienced severe nerve damage and lost feeling in his
fingers. Which of the following actions by the phlebotomist most likely led to this event?
A. The incorrect needle angle was used.
B. The needle was inserted with bevel down.
C. The needle was inserted too shallowly.
D. The cephalic vein was used.
NCCT Phlebotomy Review PG69
37. The phlebotomist has successfully found a vein and is drawing the blood. Suddenly the
blood flow stops. Which of the following is the most likely cause?
A. The tube has lost its vacuum.
B. The wrong gauge needle was used.
C. There is an occlusion in the vein.
D. The patient suffers from chronic venous insufficiency.

38. The phlebotomist is performing blood collection and an accidental arterial puncture occurs.
Which of the following actions by the phlebotomist is likely to have caused this accident? (Select
the two (2) correct answers.)
A. probing to find a vein
B. prolonged tourniquet application
C. inserting the needle near the basilic vein
D. vigorous fist pumping
E. using too large a needle

39. The phlebotomist arrives at the room of a patient on droplet precautions due to whooping
cough. Which of the following personal protective equipment must be used?
A. surgical mask
B. fluid resistant gown
C. N95 respirator
D. sterile gloves

40. In addition to standard PPE, what additional PPE should the phlebotomist do before
entering a patient's room with airborne precautions?
A. surgical mask
B. N95 respirator
C. mask with face shield
D. full face/head mask

41. Which of the following is the correct type of glove to wear when performing phlebotomy?
A. one size smaller than phlebotomist correct size
B. tight fitting gloves that are not irritable to the hands
C. loose fitting and latex free
D. one size larger than phlebotomist correct size

42. Which of the following indicates the correct order for donning PPE?
A. Gloves, mask, gown
B. Gown, mask, gloves
C. Mask, gloves, gown
D. Mask, gown, gloves
NCCT Phlebotomy Review PG70
43. Which of the following lists the correct order for putting on complete personal protective
gear?
A. gloves, gown, mask
B. gown, gloves, mask
C. mask, gloves, gown
D. gown, mask, gloves

44. Which of the following is the proper order for removing PPE?
A. gloves, gown, mask
B. gown, mask, gloves
C. gloves, mask, gown
D. mask, gloves, gown

45. Upon arriving at work the phlebotomist notices all the sharp containers are full. The
phlebotomist should
A. gently shake the containers to settle the contents.
B. seal the containers in biohazard bags and replace.
C. put the containers in a biohazard box and replace.
D. carefully empty the containers into a biohazard box and refill.

46. A phlebotomist in a hospital setting must perform a venipuncture on a patient who has been
diagnosed with varicella. Which of the following is the minimum precaution level the
phlebotomist must take?
A. isolation
B. droplet
C. airborne
D. contact

47. OSHA requires that a sharps container be


A. changed every thirty days.
B. changed when it was ¾ full.
C. placed inside a locked container.
D. placed out of reach of children.

48. A patient presents to the emergency department coughing up blood. Which of the following
safety equipment should be used to protect from droplets or airborne pathogens?
A. gloves
B. face shield
C. gown
D. mask
NCCT Phlebotomy Review PG71
49. When in the laboratory area, the phlebotomist should wear face shields when
A. receiving tests.
B. batching specimens.
C. aliquoting specimens.
D. centrifuging specimens.

50. The phlebotomist is in the process of finishing the blood draw and has untied the tourniquet
with removal of the tube from the tube holder. Upon taking the needle out of the skin, which of
the following should the phlebotomist do next?
A. Engage the safety, disassemble the tube holder with a needle, and discard.
B. Disassemble the tube holder with a needle, engage the safety, and discard.
C. Engage the safety and discard the tube holder with a needle.
D. Discard the tube holder with a needle.

51. When using a Vacutainer the needle guard should be engaged immediately
A. before applying the pressure bandage.
B. after removing the needle.
C. after removing the tourniquet.
D. before discarding in a biohazard bag.

52. A phlebotomist completes a blood collection using the evacuated system with a safety
needle. The needle safety device is activated immediately following the collection. The needle
is removed from the tube holder/adapter and discarded in a sharps container. Which of the
following correctly describes the phlebotomist’s actions?
A. The phlebotomist activated the needle safety device too soon, but removing the needle was
acceptable.
B. The phlebotomist activated the needle safety device too soon and the needle and tube
holder/adapter should not be separated.
C. The phlebotomist activated the needle safety device appropriately and removing the needle
was acceptable.
D. The phlebotomist activated the needle safety device appropriately, but the needle and tube
holder/adapter should not be separated.

53. Which of the following is the proper point at which a sharps container should be disposed
of?
A. ¾ full
B. full
C. ½ full
D. at the discretion of the facility
NCCT Phlebotomy Review PG72
Study Guide - Orders and Equipment Selection 40
Q’s
1. Sodium fluoride keeps which of the following analytes intact for 48 hours?
A. potassium
B. creatinine
C. glucose
D. cholesterol

2. Which of the following are anticoagulants found in blood collection tubes? (Select the two (2)
correct answers.)
A. sodium fluoride
B. oxalate
C. sulfate
D. heparin
E. thrombin

3. Which of the following are the additives found in a gray-top tube and what are they able to
do? (Select the three (3) correct answers.)
A. prevent coagulation
B. inhibit thrombin
C. inhibit calcium
D. prevent glycolysis
E. inhibit fibrin

5. Sodium citrate tubes have a line or an arrow somewhere on the tube. The purpose of this
mark is to
A. indicate sodium citrate levels in the tube.
B. ensure the correct blood to additive ratio.
C. allow the phlebotomist to under fill the tube and adjust blood volume.
D. indicate the correct placement of the tube in the adapter.

6. Inadequate mixing of an anticoagulant tube is likely to result in


A. hemolysis.
B. microclotting.
C. hemoconcentration.
D. no adverse effects.
NCCT Phlebotomy Review PG73
7. If an EDTA tube is underfilled, the patient’s results will likely come back with
A. a falsely high blood cell count.
B. hemoconcentration.
C. hemolysis.
D. a falsely low blood cell count.

8. A patient is at the lab for her weekly coagulation blood work. The phlebotomist should draw a
A. lavender top and fill to at least 90%.
B. light blue top and fill to at least 90%.
C. lavender top and refrigerate.
D. light blue top and refrigerate.

9. Which of the following is likely to happen to the specimen if the phlebotomist does not invert a
lavender top tube 8 to 10 times?
A. Hemolysis will occur.
B. Potassium will be released into the plasma.
C. Microclots will form.
D. Calcium will precipitate.

10. A phlebotomist collected a PT and an H&H on the dorsal side of the hand. The
phlebotomist collected a citrate tube, then an EDTA tube, and inverted each tube 3-10 times
after collecting. Once transferred to the lab, the PT specimen was rejected. The specimen was
most likely rejected due to incorrect
A. blood to additive ratio.
B. tubes collected.
C. order of draw.
D. number of inversions.

11. A patient presents with multiple skin tears and very thin skin appearance. What type of
material should the phlebotomist use following the blood draw?
A. 2 x 2 gauze and paper tape
B. 2 x 2 gauze and Band-Aid
C. 2 x 2 gauze and roll up gauze

12. Which of the following is made of a flesh colored permeable polyester fabric and acts as a
compression bandage?
A. gauze
B. Band Aides
C. Ace bandages
D. Coban
NCCT Phlebotomy Review PG74
13. A phlebotomist has just completed a successful venipuncture and is about to apply the
bandage. The patient tells the phlebotomist that he suffers from contact dermatitis. The
phlebotomist should apply a
A. standard bandage.
B. 2x2 gauze pad and paper tape.
C. 2x2 gauze pad and wrap with Coban.
D. 2x2 gauze pad with nylon tape.

14. The draw is finished and afterward the patient continues to bleed. The phlebotomist asks the
patient to hold pressure, but the patient indicates difficulty doing so. Which of the following
should the phlebotomist do next?
A. Ask the patient to hold pressure briefly until the tubes are stored.
B. Ask the patient to elevate the affected arm to stop bleeding.
C. Place a cotton ball over the area until the tubes are stored and return to hold pressure.
D. Hold pressure for the patient until bleeding is controlled.

15. Which of the following information is required on a requisition form before a phlebotomist
can send a patient’s sample to the lab? (Select the three (3) correct answers.)
A. doctor’s name
B. patient's social security number
C. patient’s date of birth
D. type of test
E. patient’s address

16. The appropriate needle gauge to use for most venipuncture collections is
A. 15.
B. 18.
C. 21.
D. 23.

17. Which of the following items is commonly found on a phlebotomy tray?


A. multi-sample needle
B. tongue depressor
C. culture swabs
D. 18-gauge needle

18. Which of the following tubes is recommended for STAT serum testing?
A. royal blue
B. pink
C. lavender
D. orange
NCCT Phlebotomy Review PG75
19. A patient arrives with orders for a complete blood count (CBC), prothrombin time (PT/INR),
and basic metabolic panel (BMP). Which of the following tubes should the phlebotomist collect?
A. lavender, blue, red
B. yellow, blue, gray
C. lavender, red, SST
D. yellow, gray, SST

20. Which needle size and type is the best choice for a patient with thin skin and extremely
fragile veins?
A. multisample needle 22g 1 1/2”
B. multisample needle 21g ¾”
C. winged infusion 23g ¾”
D. winged infusion 21g 1 ½”

21. A patient on chemotherapy has presented for routine blood work. Due to the chemotherapy,
the patient has small, weak, and frail veins. After a first failed attempt the phlebotomist
prepares to draw again. Which of the following supplies should the phlebotomist choose?
A. an ETS set
B. a butterfly with a Luer adaptor and an ETS hub
C. a 23 gauge butterfly with syringe
D. a syringe set

22. A patient receiving chemotherapy has come into the lab with an order that requires multiple
tubes. After failing on the first attempt with a butterfly, the phlebotomist is preparing for a
second attempt. Which of the following supplies should be chosen to reduce intravenous
pressure and keep the vein from collapsing?
A. a butterfly with a multisample adapter
B. a vacutainer with a 23g needle
C. a butterfly with a syringe
D. a syringe with a 23g needle

23. Physicians order an ESR and PT, PTT on an 11-month-old infant. Which of the following is
the most appropriate equipment for this blood collection?
A. Heel lancet and two microcollection tubes.
B. Heel lancet and three microcollection tubes.
C. 23G butterfly needle and two pediatric evacuated tubes.
D. 23G butterfly needle and three pediatric evacuated tubes.

24. A 78-year-old patient with fragile veins presents to the lab for recurring PT/PTT testing.
Which of the following equipment should the phlebotomist use?
A. 23g vacutainer needle, adapter, lavender tube
B. 23g vacutainer needle, adapter, light blue tube
C. 23g butterfly needle, light blue tube
D. 23g butterfly needle, lavender tube
NCCT Phlebotomy Review PG76
25. The physician has ordered a PKU test on a newborn. Which of the following should the
depth of penetration be?
A. 1.5 mm
B. 2.0 mm
C. 2.5 mm
D. 3.0 mm

26. A phlebotomist reviews a doctor’s order for a blood culture on a patient. While reviewing the
patient's chart, the phlebotomist notices that the patient has a shellfish allergy. Which sterile
solution should the phlebotomist use on this patient to avoid the potential for anaphylactic
shock?
A. 2% iodine
B. Betadine
C. Povidone iodine
D. chlorhexidine

27. After the phlebotomist puts on latex gloves and ties a non-latex tourniquet on the patient’s
arm, the patient indicates that he is highly allergic to latex. Which of the following is the
phlebotomist’s best course of action?
A. Change to non-latex gloves and proceed with the same tourniquet.
B. Change to non-latex gloves and use a new tourniquet.
C. Wash hands, change to non-latex gloves, and proceed with the same tourniquet.
D. Wash hands, change to non-latex gloves and use a new tourniquet.

28. As the phlebotomist removes the cap from the needle before a venipuncture, she notices
the needle is bent. Which of the following should the phlebotomist do?
A. File a report and return the needle to the manufacturer.
B. Inform the supervisor.
C. Recap the bent needle and put it aside.
D. Discard the needle and obtain a new one.

29. The physician has ordered a blood culture collection for a patient. Which of the following
antiseptic solutions should the phlebotomist use for cleansing the site?
A. antibacterial soap and water
B. 70% isopropyl alcohol
C. 2% hydrogen peroxide
D. 2% benzalkonium chloride

30. Which of the following is a proper antiseptic agent for drawing blood cultures?
A. chlorhexidine gluconate
B. isopropyl alcohol
C. soap and water
D. antibacterial gel
NCCT Phlebotomy Review PG77
31. Which of the following is the additive/anticoagulant in the light blue top tube?
A. EDTA
B. sodium citrate
C. lithium heparin
D. potassium oxalate

32. Which of the following tubes contains ethylenediamine tetraacetic acid (EDTA)?
A. yellow
B. royal blue
C. gray
D. red

33. A test tube with which of the following stopper colors has a glycolytic inhibitor?
A. gray
B. green
C. lavender
D. light blue

34. The physician orders a prothrombin time. Which of the following tubes should be used to
collect the specimen?
A. gray
B. light blue
C. yellow
D. red

35. Which of the following is the primary function of sodium fluoride agent found in
gray-stoppered tubes?
A. antibiotic
B. antiglycolytic
C. anticoagulant
D. antihemolytic

36. A patient has orders for a coagulation study and is on coumadin. Which of the following is
the proper additive to be added to the blood in order to complete the coagulation study?
A. EDTA
B. sodium citrate
C. heparin
D. potassium oxalate
NCCT Phlebotomy Review PG78
37. A patient presents with severe thirst, fatigue, and frequent urination. The doctor orders a
blood draw for a glucose level. To prevent glycolysis, which of the following additives is needed?
A. heparin
B. sodium fluoride
C. EDTA
D. sodium citrate

38. Which of the following additives is used with a glucose test?


A. sodium fluoride
B. EDTA
C. sodium citrate
D. thixotropic gel

39. Which of the following anticoagulant tubes also contains an antiglycolytic agent?
A. serum separator tube
B. EDTA tube
C. sodium citrate tube
D. potassium oxalate tube

40. A phlebotomist collects lavender, red (plastic), green, and light blue stopper tubes from a
patient. Which of the following lists the additives of these tubes in the correct order of draw?
A. sterile (no-additive), sodium citrate, heparin, EDTA
B. EDTA, clot activator, heparin, sodium citrate
C. sodium citrate, clot activator, heparin, EDTA
D. EDTA, sterile (no-additive), sodium citrate, heparin
NCCT Phlebotomy Review PG79
Study Guide - Patient ID and Site Preparation 39Q’s
1. A patient presents to the emergency department with a fever of unknown origin. The doctor
orders blood cultures to be drawn. Which of the following antiseptics should the venipuncture
site be cleaned with?
A. alcohol
B. Betadine
C. iodine
D. chlorhexidine

2. A person was brought to the hospital by the police to be tested for ETOH. The phlebotomist
should prep the skin by wiping the area with
A. 70% isopropyl.
B. 70% isopropyl and chlorhexidine.
C. antiseptic soap and chlorhexidine.
D. antiseptic soap.

3. In preparation for a syringe draw on a patient with no allergies, when cleaning the site the
method to use would be
A. isopropyl alcohol and clean from outside to inside in a circular motion.
B. isopropyl alcohol and clean from inside to outside in a circular motion.
C. chlorhexidine and clean from inside to outside in a circular motion.
D. chlorhexidine and clean from outside to inside in a circular motion.

4. The phlebotomist needs to draw routine labs. The skin preparation for this procedure should
be
A. Sterilize the site by using a circular motion from the center to the periphery.
B. Cleanse the site by using a circular motion from periphery to center.
C. Cleanse the site by using a circular motion from center to periphery.
D. Sterilize the site by using a circular motion from periphery to center.

5. If a larger amount of blood is needed from a child younger than two years of age, which of the
following is the best site to draw from?
A. scalp vein
B. heel
C. cephalic vein
D. basilic vein

6. While drawing a blood culture with a winged infusion set the phlebotomist should cleanse the
area with
A. isopropyl alcohol, and draw the aerobic followed by the anaerobic.
B. betadine, and draw the anaerobic followed by the aerobic.
C. isopropyl alcohol, and draw the anaerobic followed by the aerobic.
NCCT Phlebotomy Review PG80
D. betadine, and draw the aerobic followed by the anaerobic.

7. Which of the following statements reflects open-ended communication?


A. “Tell me what brings you in today.”
B. “Have you had a fever in the past 24 hours?”
C. “Are you experiencing significant pain?”
D. “Is the cough keeping you awake at night?”

8. Two patients in the waiting room with the same exact name are waiting for phlebotomy
testing. The phlebotomist should confirm patient ID by
A. asking both patients to state their name.
B. verifying spelling of the names in the medical records.
C. asking both patients to state their DOB.
D. verifying DOB in the medical records.

9. When performing a venipuncture in an outpatient setting, “person-specific” identifiers include


A. date of birth.
B. race.
C. religion.
D. medical record or ID number.

10. Which of the following must be included on a tube label? (Select the two (2) correct
answers.)
A. patient’s gender
B. patient’s name
C. phlebotomist's identification number
D. phlebotomist’s department number
E. patient’s age

11. Which of the following must the phlebotomist do to verify a hospitalized patient’s identity
before beginning a procedure?
A. Check the ID bracelet and the electronic medical record.
B. Check the ID bracelet and ask the patient to verbally confirm her identity.
C. Check the name on the room white board and the electronic medical record.
D. Check the name on the room white board and ask the patient to verbally confirm her identity.

12. According to the Joint Commission, which are the two best identifiers for a conscious
patient? (Select the two (2) correct answers.)
A. spoken name
B. identification by a family member
C. spoken social security number
D. spoken date of birth
E. room number
NCCT Phlebotomy Review PG81
13. Upon entering a patient’s room the phlebotomist informs the patient of the tests the
physician has ordered. The patient does not acknowledge the tests as the correct tests ordered.
After verifying the physician’s order, which of the following should the phlebotomist do next?
A. Ask the patient her name and continue to draw the blood.
B. Continue to draw the blood with the physician's orders.
C. Explain to the patient that the order has been verified and proceed with blood draw.
D. Ask the patient to state name and date of birth.

14. The phlebotomist enters a patient’s room, but is unable to awaken the patient. The patient
does not have a wristband on; however, there is a wrist band on the bed rail and a family
member in the room. How should the phlebotomist identify the patient?
A. Match the room number to the room number on the orders.
B. Check the wrist band attached to the bed rails.
C. Ask the family member to identify the patient.
D. Return to the lab and check the orders.

15. A phlebotomist goes to draw an inpatient. Upon identifying the patient the phlebotomist
notices the middle initial and birth year on the requisition and labels do not match the patient’s
armband. Which of the following should the phlebotomist do?
A. Go to the nurse’s station to have the information corrected.
B. Ask the nurse to verify the patient’s identity.
C. Write the correct information on the armband and in the patient’s chart.
D. Write the correct information on the requisition and labels.

16. A doctor orders a fasting urine specimen on a patient with diabetes. When should the urine
sample be collected?
A. the first urine of the morning after fasting for 8 hours
B. the second voided specimen after the fasting period
C. all voided specimens during the fasting period
D. the voided specimen after the fasting period is over and the patient has had lunch

17. A patient comes in for lab work. The orders state the patient is to have basic metabolic, lipid
and hepatic panels. After confirming the patient's identity, which of the following questions
should the phlebotomist ask?
A. What time did you eat or drink last?
B. Did you consume an alcoholic drink in the last 24 hours?
C. What medications are you currently taking?
D. When was your last blood draw?

18. Which of the following types of sample collection requires the patient to be rested?
A. timed sample
B. cortisol sample
C. stat sample
D. glucose sample
NCCT Phlebotomy Review PG82
19. Collection of a CBC specimen on a crying child can lead to an increase in which of the
following cells?
A. erythrocytes
B. platelets
C. leukocytes
D. thrombocytes

20. After collecting a lipid panel the specimen appeared milky and the results were abnormally
elevated. The phlebotomist should have verified
A. that the patient was fasting.
B. what medications the patient was taking.
C. if the patient had consumed any alcohol in the past 24 hours.
D. that the patient had no fever.

21. A lipid test has been ordered for a patient, but the patient had a cup of black tea at 0400.
Which of the following is the earliest time the test can be completed?
A. 0700
B. 1100
C. 1300
D. 1600

22. A patient presents for a CBC draw and tells the phlebotomist that she just finished a
cigarette break. Which of the following actions should the phlebotomist take?
A. Proceed with the draw.
B. Make a note on the requisition form and proceed with the draw.
C. Consult with the nurse before conducting the draw.
D. Ask the patient to reschedule and to refrain from smoking 12 hours before the draw.

23. Which of the following effects does warming the site have on venipuncture?
A. It allows the veins to roll.
B. It prevents hemoconcentration.
C. It makes blood flow more quickly.
D. It increases localized blood flow.

24. Leaving a tourniquet in place longer than one minute is likely to cause
A. hematuria.
B. hematoma.
C. thrombosis.
D. hemoconcentration.
NCCT Phlebotomy Review PG83
25. A phlebotomist is struggling with finding a vein on a patient. Which of the following is
appropriate for venous distention?
A. Have the patient pump a fist to make the vein more prominent.
B. Have the patient pump a fist and release as the phlebotomist inserts the needle.
C. Apply a warm compress for about 3-5 minutes.
D. Apply a cool compress for about 3-5 minutes.

26. Which of the following is likely to cause erroneous results with cell counts, coagulation
studies, and protein levels?
A. leaving the tourniquet on longer than two minutes
B. having the patient make a fist
C. the patients’ hand falls asleep while opening and closing the fist
D. releasing the tourniquet before all blood tubes are filled

27. Which of the following blood collection tube parameters should be verified to help prevent
pre-analytical error? (Select the three (3) correct answers.)
A. type
B. manufacture date
C. expiration date
D. temperature
E. size

28. An elderly patient is seen in the lab for a routine blood draw and presents with bruises in the
antecubital area of both arms. Which of the following locations should the phlebotomist use to
draw the sample?
A. just below the hematoma in either arm
B. proximal to the hematoma
C. any area surrounding the hematoma
D. dorsal hand vein

29. When selecting a site for venipuncture the phlebotomist should avoid any area with which of
the following? (Select the three (3) correct answers.)
A. tattoos
B. rash
C. previous venipuncture sites
D. edema
E. scarring

30. The phlebotomist checks both arms and hands, but cannot find a vein. Which of the
following should the phlebotomist do next?
A. Check the patient's feet for a vein.
B. Perform a capillary puncture.
C. Remove the fingertips of the gloves and recheck them.
D. Ask a coworker for help.
NCCT Phlebotomy Review PG84
31. A pediatrician has ordered a lead screening on a toddler. Which of the following is the most
appropriate site for a capillary blood collection?
A. heel stick
B. thumb (first digit)
C. pointer finger (second digit)
D. ring finger (fourth digit

32. When performing a capillary stick on an infant, the phlebotomist should use the
A. Central arch area of the heel.
B. inside of the heel.
C. lateral side of the heel.
D. center portion of the heel.

33. A 2-week-old infant has been brought in the lab for a repeat PKU/neonatal screening. Which
of the following collection procedures is appropriate to collect the specimens?
A. venipuncture using evacuated tube system
B. venipuncture using winged infusion set
C. capillary using dermal puncture lancet
D. capillary using heel puncture lancet

34. Skin puncture in adults is most often performed using the distal portion of the
A. 4th or 5th fingers.
B. 3rd or 4th fingers.
C. index finger.
D. 2nd or 3rd fingers.

35. Which of the following is an appropriate capillary puncture site on a 15-month-old patient?
A. medial plantar heel
B. lateral plantar heel
C. distal segment of the middle finger
D. distal segment of the little finger

36. When performing a capillary puncture on an infant, which of the following is the preferred
site?
A. posterior curvature of the heel
B. lateral or medial plantar surface of the heel
C. ring finger
D. index finger
NCCT Phlebotomy Review PG85
37. Which of the following describes proper aseptic technique in the cleaning of the
venipuncture site?
A. Let the alcohol air dry before inserting the needle.
B. Allow the alcohol to soak into the skin, then wipe off excess with sterile gauze.
C. Wipe the site in a concentric motion with an alcohol wipe and insert the needle while wet.
D. Wave above the site with a gloved hand to dry the alcohol faster.

38. When preparing a site with alcohol for venipuncture, the phlebotomist should cleanse in
which of the following motions?
A. lightly from the inside out in an oval motion
B. in a light zig zag pattern
C. back-and-forth friction, in a horizontal or vertical motion
D. concentric circular friction from the outside inward

39. An 86-year-old patient presents with thin papery skin. Which of the following should the
phlebotomist do when applying the tourniquet?
A. Apply the tourniquet directly above the chosen site.
B. Wrap the patient's arm with gauze and apply the tourniquet.
C. Apply the tourniquet with less pressure.
D. Use a cloth tourniquet.
NCCT Phlebotomy Review PG86
Study Guide - Problems and Correction 40Q’s
1. An ESRD patient has an IV in the left forearm and an AV fistula in the right arm. The
phlebotomist used an ETS to collect from the left antecubital area and there was an error in the
results. The phlebotomist should have collected
A. above the fistula in the right arm.
B. below the fistula in the right arm.
C. below the IV in the left arm.
D. above the IV in the left arm.

2. In which of the following time frames should a tourniquet be routinely released on a patient
while performing a blood draw?
A. within 30 seconds
B. within one minute
C. within three minutes
D. within five minutes

3. After cleansing the site for a venipuncture with alcohol, it should be allowed to completely dry
to avoid
A. hemoconcentration.
B. hemolysis.
C. hemostasis.
D. hemodilution.

4. Why should the phlebotomist avoid collecting a CBC from a crying infant?
A. The specimen may be hemolyzed.
B. WBC’s may be elevated.
C. The specimen may be hemoconcentration.
D. Platelets are more likely to clump.

5. Which of the following tubes must be filled to at least 90% for a single test?
A. green
B. pink
C. light blue
D. lavender

6. When a blood sample does not meet sample size criteria, it is considered to be which of the
following?
A. QA
B. QC
C. hemolyzed
D. QNS
NCCT Phlebotomy Review PG87
7. Which of the following variables affect glucose strips while stored in the container?
A. excessive heat and moisture
B. excessive heat and bright light
C. moisture and being shaken
D. bright light and being shaken

8. A phlebotomist collected a serum potassium specimen from a small antecubital vein using a
21 gauge butterfly in a SST tube. The specimen was determined to be unsuitable for testing.
The specimen was most likely rejected because it was
A. QNS.
B. hemolyzed.
C. exposed to light.
D. collected in the wrong tube.

9. After drawing an SST tube with a winged infusion set using a syringe adaptor, the
phlebotomist waits 20 minutes and centrifuges the specimen. The laboratory later notifies the
phlebotomist that the serum sample was pink. Which of the following is likely to have caused
this?
A. Oxygen was introduced to the sample during transfer.
B. The blood was not allowed to clot properly.
C. The needle gauge was too small.
D. The tube was underfilled.

10. A chemotherapy patient comes to have blood drawn and the phlebotomist notices that his
veins are thin and fragile. The choice of equipment should be
A. capillary tube.
B. vacutainer.
C. syringe.
D. lancet.

11. Which of the following kinds of tourniquet should be used for an obese patient?
A. latex
B. velcro
C. nitrile
D. BP cuff

12. The patient has an IV on the right arm and sclerosed veins on the left arm. Which of the
following sites should the phlebotomist attempt to draw from first?
A. above the IV site
B. below the IV site
C. left arm
D. left hand
NCCT Phlebotomy Review PG88
13. How should the phlebotomist reposition the needle if he misses the vein?
A. Move the needle slightly forward or back.
B. Reposition and probe for the vein.
C. Push the needle side to side.
D. Remove the needle and start over.

14. After puncturing the skin for a glucose test, the phlebotomist has trouble obtaining blood
from the puncture site. Which of the following should the phlebotomist do next?
A. Re-stick at the same site.
B. Scrape the test strip against the puncture site to obtain the required sample.
C. Apply pressure, bandage the site, and perform a second skin puncture.
D. Gently squeeze and release the finger.

15. While drawing blood on an 80-year-old patient, the phlebotomist notices that blood has
stopped entering the vacutainer and the vein has collapsed. Which of the following should the
phlebotomist do next?
A. Remove the tourniquet, pull out the needle, and select a different vein.
B. Pull the needle back and reposition into the vein.
C. Feel for the vein, reposition the needle, and enter the vein.
D. Advance the needle deeper into the vein.

16. A phlebotomist begins to collect a blood specimen from a prominent antecubital vein. When
the tube is engaged, a small amount of blood spurts into the tube and then stops. Which of the
following is most likely to have occured?
A. The bevel has gone through the vein.
B. The bevel is partially out of the skin.
C. The vein has collapsed.
D. The tube has expired.

17. A phlebotomist performs a venipuncture on a dorsal hand vein of an adult using a 23g
butterfly needle and evacuated tubes for PT and CBC. The evacuated tube begins to fill and
then stops abruptly due to a collapsed vein. The phlebotomist should recollect using
A. a 25g needle.
B. small volume tubes.
C. capillary puncture.
D. a 22g needle.

18. When collecting two tubes for a blood sample, the first tube fills with blood, but the second
tube does not return any blood. Which of the following should the phlebotomist do first to
complete the blood draw?
A. Palpate the vein to verify placement.
B. Discontinue and attempt to draw from another site.
C. Remove the tourniquet.
D. Change the blood tube.
NCCT Phlebotomy Review PG89
19. A 14-month-old pediatric patient is having a sample collected from the vein, but the blood
has stopped flowing. Which of the following actions should the phlebotomist take to complete
the draw? (Select the three (3) correct answers.)
A. Sample from the dorsal hand vein.
B. Sample from the lateral portion of the infant’s heel.
C. Draw a capillary sample from a finger.
D. Release the tourniquet.
E. Replace the tube.

20. The phlebotomist is in the process of filling a second tube, but the tube is not filling. The
needle did not move during tube switch, the vein did not roll, and the first tube filled without any
problems. Which of the following actions should the phlebotomist take next?
A. End the draw and make a second attempt.
B. Remove the existing tube and try another tube of the same type.
C. Remove the existing tube and continue with the order of draw.
D. Advise the physician that the requisition cannot be completed.

21. While performing a venipuncture, the phlebotomist gets a tiny amount of blood and then
nothing else. A hematoma starts to develop. Which of the following is the phlebotomist’s best
course of action?
A. Gently pull the needle back and continue the blood draw.
B. Gently push the needle further into the vein and continue the blood draw.
C. Release tourniquet and stop the procedure.
D. Turn the needle since the bevel is down.

22. Which of the following is a likely complication from repeated phlebotomy procedures in the
same area?
A. thrombosis
B. sclerosed veins
C. petechiae
D. varicose veins

23. Small red dots on a patient's skin due to a blood clotting abnormality are referred to as
A. purpura.
B. petechiae.
C. hemolysis.
D. hematoma.
NCCT Phlebotomy Review PG90
24. The doctor has ordered a dermal puncture on an infant. During the draw the infant suddenly
moves and the phlebotomist punctures the calcaneus. Which of the following complications is
likely to arise from this incident?
A. petechiae
B. Achilles tendon rupture
C. osteomyelitis
D. osteotome

25. While finishing a draw on an outpatient with diabetes, the patient starts to complain of
lightheadedness. Which of the following should the phlebotomist do next?
A. Check the patient’s glucose level.
B. Lay the patient on the floor.
C. Give the patient orange juice.
D. Call for assistance.

26. A phlebotomist is drawing blood on a geriatric patient. When completing the draw, the
phlebotomist notices the formation of a hematoma. Which of the following should the
phlebotomist do next?
A. Instruct the patient not to use the venipuncture arm for any heavy lifting.
B. Hold the patient’s arm in a downward position and apply ice.
C. Apply firm pressure at the site for at least five minutes.
D. Have the patient lie down on an exam table.

27. In a hospital setting a phlebotomist has just finished filling their last tube for collection. After
releasing the tourniquet the phlebotomist accidently rotates the needle at a 45 degree angle
from the insertion point while it is still in the arm. Which of the following might occur?
A. hemolysis
B. bruising
C. petechiae
D. nothing

28. The phlebotomist is collecting a CBC from a patient’s hand using the syringe method. There
is no blood return in the hub and the patient indicates pain in the area. Which of the following
should the phlebotomist do next?
A. Advance the needle a little further and observe for blood flash in the hub.
B. Release the tourniquet and discontinue the draw.
C. Pull the needle back slightly.
D. Reposition the needle and release the tourniquet.
NCCT Phlebotomy Review PG91
29. During a pediatric blood draw, the parent who is holding the child suddenly allows
movement of the arm being used for the blood draw. Which of the following actions should the
phlebotomist take?
A. Ask the parent to regain control of the child's arm.
B. Ask the parent to leave.
C. Remove the needle immediately.
D. Call for help.

30. The patient has a hematoma at the site the phlebotomist performed the venipuncture. Which
of the following are likely causes of the hematoma? (Select the three (3) correct answers.)
A. failure to remove tourniquet prior to removing the needle
B. insertion of the needle through the vein
C. undisclosed antihypertensive medication
D. excessive probing to locate the vein
E. wrong needle gauge used for venipuncture

31. After applying the tourniquet on a patient, the phlebotomist noticed the development of
petechiae. Which of the following should the phlebotomist do next?
A. Remove the tourniquet and assess the other arm.
B. Continue with the procedure.
C. Consult the nurse.
D. Refer the patient back to the physician.

32. While the phlebotomist is drawing the first of five tubes the patient becomes pale and begins
to sweat. Which of the following should the phlebotomist do first?
A. Reassure the patient.
B. Discontinue the draw.
C. Pause the collection.
D. Call for assistance.

33. A patient on aspirin therapy is seen for routine blood draw. Which of the following should the
phlebotomist do to control bleeding after the procedure?
A. Maintain pressure for at least one minute.
B. Allow the patient to maintain pressure until the bleeding has stopped.
C. Maintain pressure for five minutes
D. Maintain pressure until the bleeding stops.

34. Which of the following should the phlebotomist do for a patient on anticoagulant therapy to
stop bleeding?
A. Make sure adequate pressure is held over site until bleeding stops.
B. Hold very tight pressure over site until bleeding stops.
C. Apply a pressure bandage until bleeding stops.
D. Have the patient apply pressure.
NCCT Phlebotomy Review PG92
35. The patient has been taking blood thinners for some time. The phlebotomist asks the patient
to hold pressure on the site after the needle is removed. As the phlebotomist is labeling the
tubes, he notices that the gauze at the site is soaked in blood. Which of the following should the
phlebotomist do first?
A. Apply an ice pack.
B. Call for help.
C. Replace the gauze and hold pressure.
D. Elevate the arm and apply a pressure bandage.

36. Which of the following special precautions should be taken when drawing blood samples
from a patient on heparin therapy?
A. Use light to moderate tourniquet pressure.
B. Apply site pressure for a prolonged time.
C. Avoid using the basilic vein.
D. Avoid using 21 gauge needles.

37. Following an uncomplicated venipuncture, the patient is still bleeding after three minutes of
continuous pressure by the phlebotomist. The phlebotomist asks the patient if she is on
anticoagulants. The patient indicates that she is, and states that this is normal for her. How
should the phlebotomist proceed?
A. Apply a pressure bandage and let the patient leave.
B. Bend the arm up, apply a pressure bandage, wait two more minutes, and notify the nurse.
C. Hold direct pressure for five more minutes, apply a bandage, and let the patient leave.
D. Hold direct pressure until the bleeding stops, apply a bandage, and notify the nurse.

38. A patient is scheduled to have her blood collected to evaluate Coumadin (warfarin) therapy.
Which of the following actions should the phlebotomist take when performing a venipuncture on
this patient? (Select the three (3) correct answers.)
A. Collect PTT in a light blue tube.
B. Collect PT/INR in a light blue tube.
C. Apply firm and direct pressure to the venipuncture site for at least 5 minutes.
D. Inform the patient about possible hematoma formation.
E. Apply a pressure bandage immediately after venipuncture.

39. The medical record number and patient’s date of birth on the ID band matches requisition,
but the patient’s name is spelled differently on the ID band. Which of the following should the
phlebotomist do?
A. Have the patient write the correct spelling of his name on the requisition.
B. Draw the specimen because the medical record number matches.
C. Collect the specimen and report the error to the patient’s nurse.
D. Do not collect the specimen until the difference is resolved.
NCCT Phlebotomy Review PG93
40. The phlebotomist realizes after finishing the collection that she has missed a tube needed
for testing. Which of the following corrective actions should the phlebotomist take?
A. Transfer blood from a full tube into the missing tube.
B. Inform the patient and redraw all the tubes.
C. Inform the patient and perform a 2nd collection of the missing tube.
D. Have the patient reschedule to draw the missing tube.
NCCT Phlebotomy Review PG94
Study Guide - Quality and Professional Issues 42Q’s
1. The hospital is short staffed and has asked the phlebotomist on duty to help. Which of the
following tasks are considered outside the normal phlebotomist scope of practice?
A. Collect a tissue sample.
B. Process venous samples.
C. Perform a heel stick on a 2-day-old infant.
D. Perform point-of-care tests.

2. Which of the following is within the scope of the phlebotomist’s duties?


A. Obtaining specimens from IV sites.
B. Calling critical values to providers.
C. POCT.
D. Correction of test orders.

3. A phlebotomist collects blood from a postoperative patient in a hospital bed with no problems.
Before the phlebotomist leaves, the patient asks for help moving from the bed to a chair. While
the phlebotomist is assisting the patient, he falls. Was the phlebotomist wrong for helping the
patient?
A. Yes, because it was out of the phlebotomist’s scope of practice.
B. Yes, because there should be more than one person moving a patient from a bed.
C. No, because the phlebotomist’s actions are covered under the Joint Commission standards.
D. No, because the patient has a right to receive help.

4. Which of the following is the unique number assigned to a specimen request?


A. health facility number
B. DOB
C. patient ID
D. accession number

5. Which of the following is a CLSI requirement for every specimen label?


A. patient SSN
B. patient diagnosis
C. physician’s name
D. unique patient identifier

6. When a test order is entered into the laboratory information system (LIS), labels are
generated for each tube to be drawn. A unique number is shown on each label. What is the
name of that number?
NCCT Phlebotomy Review PG95
A. medical record number
B. FIN number
C. accession number
D. container identification number (CID)

7. After a blood test is drawn, the phlebotomist goes to a computer station to “verify” the draw.
The purpose of this step is to
A. tell the doctor that the test is completed.
B. document the site of the draw and the tubes used.
C. tell the billing department to charge for the test.
D. tell the medical technologist in the lab to perform the test.

8. A phlebotomist has entered a test order into the LIS, generates a set of tube labels, but
forgets to sign out of the system when she goes to draw the blood. Which of the following is
likely to happen because of this action?
A. The test order will not be transmitted to the laboratory instruments.
B. All subsequent orders will be entered under her password and initials.
C. The results will not be available to the nursing unit.

9. A patient asks the phlebotomist to forward the results of the tests to her PCP. This action is
covered under
A. implied consent.
B. written consent.
C. HIPAA.
D. CLIA.

10. Which of the following is considered PHI? (Select the three (3) correct answers.)
A. diagnosis code
B. year of admission
C. date of birth
D. hospital name
E. phone number

11. Which of the following actions ensures compliance with HIPAA standards?
A. Billing only for services performed.
B. Explaining the procedure to the patient.
C. Properly drawing the patient.
D. Addressing patient complaints in private.

12. A woman calls the lab and asks if her husband has been in to get his blood drawn yet. She
gives the phlebotomist his full name and his birthdate. Which of the following should the
phlebotomist do?
A. Let her know if her husband has been in already.
B. Let her know the information can’t be released over the phone.
NCCT Phlebotomy Review PG96
C. Take her name and number and have a nurse call her back.
D. Transfer her call to the integrity officer of the hospital.

13. A phlebotomist has a requisition to collect from a patient who is a neighbor. After work, the
phlebotomist calls the homeowners association to let them know that a fellow neighbor is in the
hospital so that they will send flowers. The phlebotomist did not disclose why the neighbor was
in the hospital or the neighbor’s room number. Did the phlebotomist act properly?
A. Yes, because the phlebotomist did not violate HIPAA.
B. Yes, because the phlebotomist did not disclose any PHIs.
C. No, because the phlebotomist breached confidentiality.
D. No, because the phlebotomist was negligent.

14. Repeated clerical or technical errors by a phlebotomist are considered


A. assault.
B. fraud.
C. liability.
D. negligence.

15. Which of the following is an error a phlebotomist might commit that is most likely to lead to a
sentinel event?
A. failing to document QC
B. using the wrong gauge needle
C. misidentifying a patient
D. using the wrong ETS tube

16. The main reason a phlebotomist should avoid a hematoma for a blood collection site is
A. it will cause the patient pain.
B. it could cause erroneous lab results.
C. the blood has started to clot and therefore is hard to draw.
D. A previous draw has been performed on that spot.

17. While documenting quality control for a new sample tube lot number it is discovered to be
expired. Which of the following is likely to occur if the tube is used? (Select the three (3) correct
answers.)
A. The tube will not fill completely.
B. The tube may give questionable results.
C. The tube may infect the patient.
D. The additive may not work.
E. The patient’s vein may collapse.

18. After processing samples from venous blood, the phlebotomist finds that the volume of the
separated serum/plasma is significantly less than normally recovered. The phlebotomist should
next check the
A. centrifuge.
NCCT Phlebotomy Review PG97
B. patient’s blood type.
C. needles & blood collection tubes.
D. temperature.

19. Which of the following should be done to maintain the quality of specimens sent to the
laboratory for diagnostic testing?
A. Use a timer for serum clotting and use a timed centrifuge.
B. Record the temperature of the blood and record the temperature of the lab.
C. Use a timed centrifuge and record the temperature of the blood.
D. Record the temperature of the lab and use a timer for serum clotting.

20. Which of the following are quality indicators pertaining to phlebotomy? (Select the two (2)
correct answers.)
A. number of needlestick injuries
B. blood culture contamination rates
C. performance of proficiency testing
D. number of CAP inspection deficiencies
E. the number of butterfly sets used per month

21. When performing quality control tests in the laboratory, the phlebotomist must
A. ensure the specimen is collected properly.
B. document results in the log.
C. document results on the requisition form.
D. document results on the progress report.

22. In an outpatient setting, temperature checks for refrigerated instruments should be recorded
A. daily on a log sheet.
B. per manufacturer’s instructions.
C. if temp is out of range.
D. using external liquid controls.

23. Which of the following storage conditions should be verified when restocking blood
collection tubes in the laboratory?
A. type & quantity
B. quantity & expiration
C. expiration & temperature
D. temperature & type

24. A phlebotomist notices an entry on the temperature log that is higher than the indicated
range. Which of the following should the phlebotomist do next?
A. Ask the patients if it is too warm.
B. Report to a supervisor.
C. Adjust the thermostat.
D. Update the entry into the correct range.
NCCT Phlebotomy Review PG98
25. A phlebotomist is asked to perform a pregnancy test on a urine sample. The test result is
positive but the QC indicator line does not develop. Which of the following should the
phlebotomist do?
A. Report the test as positive and document that the QC was not functional.
B. Report the test as negative because the QC was not functional.
C. Obtain another pregnancy test cartridge and repeat the test.
D. Ask for a new urine specimen.

26. The phlebotomists in an outpatient area record the temperatures of their specimen storage
refrigerator on log sheets every day. A staff member notices that the readings have not been
written down for the last two days. Which of the following is the likely consequence of this
omission?
A. The phlebotomist who neglected to document the temperatures will be fired.
B. An inspector will give the lab a deficiency on the next inspection.
C. The readings will be filled in by the supervisor.
D. Patient results will be compromised.

27. According to the Patient’s Bill of Rights, which of the following phlebotomist actions indicates
a need for more professionalism?
A. Performing a venipuncture on the basilic vein.
B. Obtaining implied consent from the patient.
C. Discussing the lab results with the patient.
D. Drawing a child with assent.

28. A hospital patient states that he does not want his blood drawn. The phlebotomist tells the
patient, “if you don’t let me collect your blood, your illness will become more critical.” This
statement is considered which of the following?
A. battery.
B. libel.
C. tort.
D. assault.

29. The phlebotomist has an order to draw a CBC on an inpatient. After entering the room, the
phlebotomist sees that the patient is sleeping. Which of the following actions should the
phlebotomist take next?
A. Perform the draw without waking the patient.
B. Identify yourself and describe the procedure.
C. Attempt to gently wake the patient.
D. Consult the nurse.
NCCT Phlebotomy Review PG99
30. When dealing with elderly patients, which of the following can a phlebotomist do to improve
the communication process? (Select the three (3) correct answers.)
A. speak softly
B. speak slowly and loudly
C. avoid visual aids
D. make eye contact
E. take time and listen carefully

31. The phlebotomist needs to collect a blood sample from a geriatric patient. The patient’s
veins are visible and small, and the skin is translucent. Which of the following should the
phlebotomist use? (Select the three (3) correct answers.)
A. ETS multi-sample needle
B. Band-Aid
C. pediatric tubes (short draw tubes)
D. butterfly
E. Coban

32. Which of the following is the key element in effective communication?


A. eye contact
B. personal space
C. active listening
D. body language

33. The phlebotomist enters a room with airborne precautions and asks an elderly patient to
verbally state her name and date of birth. The patient states she can’t hear what the
phlebotomist is saying. Which of the following should the phlebotomist do?
A. Move closer and speak louder so the patient can hear through the N95 respirator.
B. Briefly pull the N95 respirator away from the face so the patient can hear.
C. Continue with the procedure and draw the patient based on implied consent.
D. Shout through the N95 respirator so the patient can hear.

34. Quality controls are performed on CLIA waived tests to


A. Ensure lot numbers are correct.
B. identify the correct manufacturer.
C. prevent testing errors.
D. prevent cross-contamination.

35. Which of the following phlebotomist actions is contraindicated by the CLSI the Procedure
Manual for Blood Draws?
A. Use a blood pressure cuff in place of a tourniquet.
B. Heat the AC area to make veins more prominent.
C. Gently slap the area to make veins more prominent.
D. Hang the arm down for several seconds to have more blood pool in the arm.
NCCT Phlebotomy Review PG100
36. A glucose specimen in a sodium fluoride tube was forgotten on a phlebotomist’s tray and
wasn’t discovered until 6 hours later by the phlebotomist on the next shift. Should the specimen
be submitted for testing or should it be recollected?
A. Yes, glucose specimens collected in gray top tubes are stable for 24 hours at room temp.
B. Yes, glucose specimens collected in gray top tubes are stable for 48 hours at room temp.
C. No, glucose specimens collected in gel barrier tubes must be centrifuged within 1 hour.
D. No, glucose specimens collected in gel barrier tubes must be centrifuged within 30 min.

37. The phlebotomist was asked to perform a UA and to perform microscopic testing of the
specimen. Which of the following standards does this action violate?
A. OSHA
B. CLIA
C. CLSI
D. COLA

38. Which of the following are National Patient Safety Goals? (Select the three (3) correct
answers.)
A. Speaking directly to a patient in a calm and friendly way.
B. Identify the patients correctly.
C. Ask about latex or alcohol allergies before beginning procedures.
D. Improve staff communication.
E. Prevent infection.

39. A phlebotomist has drawn samples on a suspected DUI driver. Which of the following should
occur when giving the sample to the lab?
A. The sample identification is verified.
B. A receipt is signed and dated.
C. The sample is photographed.
D. The lab technician verifies the search warrant.

40. To avoid collapsing the vein during blood draw using the syringe method, the phlebotomist
should
A. pull the plunger steadily.
B. quickly withdraw the needle when the procedure is complete.
C. use the thumb to apply firm pressure.
D. use gentle pressure when inserting the needle.

41. Before performing a venipuncture on a conscious patient, the most effective way to identify
the patient is to
A. match their wrist band to the requisition form.
B. ask the patient for his name and qualifying information.
C. check the requisition form with the chart.
D. ask the patient to complete the identification portion of the requisition form.
NCCT Phlebotomy Review PG101
42. Which of the following should be on the label of a specimen after the venipuncture? (Select
the three (3) correct answers.)
A. phlebotomist’s initials
B. patient’s D.O.B.
C. ICD-10 diagnosis code
D. date of draw
E. lab ID
NCCT Phlebotomy Review PG102
13 Case Scenarios
Case Scenario 1
Objective: Infection Control, Standard Precautions and Safety
Case: You work in a hospital and your first draw of the day is on a patient known to have active
tuberculosis. This patient does not appear to be ill, but coughs a lot. A sign on the patient's door
indicates airborne precautions.

1. Explain the significance of airborne precautions: ___________________________________


____________________________________________________________________________
____________________________________________________________________________

2. How should you proceed? _____________________________________________________


____________________________________________________________________________
____________________________________________________________________________

3. What precautions should you take during the draw? _________________________________


____________________________________________________________________________
____________________________________________________________________________

4. What precautions should you take when handling the sample? ________________________
____________________________________________________________________________
____________________________________________________________________________

5. What precautions should you take when handling the sample? ________________________
____________________________________________________________________________
____________________________________________________________________________

Case Scenario 2
Objective: Anatomy and Physiology of Circulatory System
Case: You are called to the Emergency Department to draw a STAT CBC, LYTES, and BILI from
a 33-year-old male who appears very ill. You notice that the whites of his eyes are almost
yellow, as is his skin tone, and that he is very weak. He has an IV running in his left hand.
1. Where do you look first to find a vein? ___________________________________________
____________________________________________________________________________
____________________________________________________________________________

2. What tubes do you draw? _____________________________________________________


____________________________________________________________________________
____________________________________________________________________________

3. What do these tests typically check? ____________________________________________


____________________________________________________________________________
____________________________________________________________________________
NCCT Phlebotomy Review PG103
4. What diagnosis might be among those the physician is considering? ___________________
____________________________________________________________________________
____________________________________________________________________________

5. What might these combinations of laboratory tests typically check: Creatinine, BUN, routine
urinalysis? ___________________________________________________________________
____________________________________________________________________________
____________________________________________________________________________

6. What might these combinations of laboratory tests typically check: PTT, APTT, ProTime,
Platelet Count? _______________________________________________________________
____________________________________________________________________________
____________________________________________________________________________

7. What might these combinations of laboratory tests typically check: Alkaline Phosphatase,
Calcium, LD? _________________________________________________________________
____________________________________________________________________________
____________________________________________________________________________

8. What might these combinations of laboratory tests typically check: Hemoglobin and
Hematocrit? _________________________________________________________________
____________________________________________________________________________
____________________________________________________________________________

9. What might these combinations of laboratory tests typically check: Troponin and CK-MB? ___
____________________________________________________________________________
____________________________________________________________________________

Case Scenario 3
Objective: Major Body Systems
Case: Your grandfather has just returned from the physician’s office and is trying to remember
what the physician told him. He went to the doctor because he was coughing up blood. He tells
you that the doctor is doing a battery of tests on one of his major body systems, but that it is a
long word and he can’t quite remember which one it is. He says it has something to do with his
breathing.

1. Which body system does he mean? _____________________________________________


____________________________________________________________________________
____________________________________________________________________________

2. What other major systems are there? ____________________________________________


____________________________________________________________________________
____________________________________________________________________________
NCCT Phlebotomy Review PG104
3. Which veins of the circulatory system are most commonly used for venipuncture? ________
____________________________________________________________________________
____________________________________________________________________________

4. Which finger(s) is/are most commonly used for microsampling? _______________________


____________________________________________________________________________
____________________________________________________________________________

5. What part of an infant foot is used for microsampling? _______________________________


____________________________________________________________________________
____________________________________________________________________________

6. What type of patient can NOT be drawn in the leg or foot? ___________________________
____________________________________________________________________________
____________________________________________________________________________

Case Scenario 4
Objective: Proper Identification of Patients and Samples & Importance of Accuracy in Patient
Care
Case: Your fellow phlebotomist brings in a serum tube that he has just collected and labeled as
“John Jones”. You have been faxed one set of lab test orders for a glucose test on John H.
Jones and another set of lab orders for a calcium level on John M. Jones. You ask your friend
which Jones he drew. He says that it doesn’t really matter, since both need serum tubes
anyway. This is the label on the tube:

1. Is your friend correct that it doesn’t matter which tube is sent to the lab? _________________
____________________________________________________________________________
____________________________________________________________________________

2. What is your next step? _______________________________________________________


____________________________________________________________________________
____________________________________________________________________________
NCCT Phlebotomy Review PG105
3. What is often considered the single most important step in blood collection? _____________
____________________________________________________________________________
____________________________________________________________________________

4. What could have been done to prevent a patient and specimen identification mix-up error? _
____________________________________________________________________________
____________________________________________________________________________

5. What other forms of numeric identifiers can be used? _______________________________


____________________________________________________________________________
____________________________________________________________________________

Case Scenario 5
Objective: Proper Selection and Preparation of Skin Puncture Sites, including Selection of
Disinfectants
Case: You will be performing venipunctures on three patients in a row. The first patient has
orders for electrolytes only. The second patient has a blood culture ordered. Your last patient
has a CBC ordered. You wash your hands and put on new gloves before drawing your first
patient. You complete the blood draw. You take off your gloves, fill out some paperwork, and
then go back to draw your second patient’s blood work. You grab a new pair of gloves and
perform the blood culture draw. You discard the gloves when finished and move toward your
third and final patient. After greeting her, you look for another new pair of gloves.

1. How are you doing so far with respect to following Standard Precautions? _______________
____________________________________________________________________________
____________________________________________________________________________

2. What should you do before drawing patient number three? ___________________________


____________________________________________________________________________
____________________________________________________________________________

3. How would cleansing the draw site differ between patients 1 and 2? ____________________
____________________________________________________________________________
____________________________________________________________________________

4. What are Standard Precautions? _______________________________________________


____________________________________________________________________________
____________________________________________________________________________

5. What color tube tops are generally drawn for cultures? ______________________________
____________________________________________________________________________
____________________________________________________________________________
NCCT Phlebotomy Review PG106
Case Scenario 6
Objective: Blood Collection Equipment; Types of Tubes; Additives; Order of Draw; Special
Precautions
Case: Here is a partial list of equipment on your drawing tray:
● 21 gauge needles (for syringes and multisample adapters)
● 23 gauge needles (for syringes and multisample adapters)
● Microsampling lancets
● Adult size red and speckled tops, lavender top, light blue top, and green top
tubes
● Small volume red and lavender containers
● Pedi-red and Pedi-lavender top tubes
● Assorted microcollection containers

1. What would you consider using to draw a CBC order on a 5-year-old? __________________
____________________________________________________________________________
____________________________________________________________________________

2. What would you consider using to draw a creatinine on a 45-year-old male with large veins?
____________________________________________________________________________
____________________________________________________________________________

3. What would you consider using to draw blood from an infant heel? _____________________
____________________________________________________________________________
____________________________________________________________________________

4. What are the medical terms for the smallest veins and arteries? _______________________
____________________________________________________________________________
____________________________________________________________________________

5. What happens if you use a drawing device with excessive vacuum on a patient with a very
thin and fragile vein? ___________________________________________________________
____________________________________________________________________________
____________________________________________________________________________

6. If you had to draw both the red and lavender tops on a single patient, which would you draw
first? What is the order of draw for all tube top colors? _________________________________
____________________________________________________________________________
____________________________________________________________________________

7. What additives are in which tubes? ______________________________________________


____________________________________________________________________________
____________________________________________________________________________
NCCT Phlebotomy Review PG107
Case Scenario 7
Objective: Appropriate Disposal of Sharps, Needles & Waste
Case: After drawing a blood sample on Mrs. Baxter in the hospital, you discard your needle in
an approved sharps container on your tray and finish up your paperwork. As you are about to
leave the room, you notice a capped needle on the floor next to Mrs. Baxter’s bed, some
obviously “used” gloves (turned inside out) close by, and some liquid spilled on the floor. It looks
like water but you aren’t sure.

1. Describe your course of action. _________________________________________________


____________________________________________________________________________
____________________________________________________________________________

2. Containers that are used to dispose of sharps should have what characteristics? __________
____________________________________________________________________________
____________________________________________________________________________

3. If you accidentally stick yourself with the needle as you pick it up, how would you clean your
wound? _____________________________________________________________________
____________________________________________________________________________
____________________________________________________________________________

4. Can you name at least three laws or regulations that govern safe practices for phlebotomy?
What do they address? _________________________________________________________
____________________________________________________________________________
____________________________________________________________________________

Case Scenario 8
Objective: Anticoagulation and Coagulation
Case: Mrs. Bledsoe enters the hospital with orders for Prothrombin Time and Partial
Thromboplastin Time tests. As the blue top tube is filling with blood, the physician enters the
room and asks also that you draw a CBC, DIFF, and Type and Crossmatch.

1. What tube or tubes were you originally drawing for the PT and PTT? ___________________
____________________________________________________________________________
____________________________________________________________________________

2. What tubes would be needed for the new requests? ________________________________


____________________________________________________________________________
____________________________________________________________________________

3. Can they be added to the end of the first draw without adversely affecting test results? _____
____________________________________________________________________________
____________________________________________________________________________
NCCT Phlebotomy Review PG108
4. Why would a Type and Crossmatch be ordered? ___________________________________
____________________________________________________________________________
____________________________________________________________________________

5. Can you explain why a patient’s blood does not normally clot as it is flowing through the
patient’s bloodstream? _________________________________________________________
____________________________________________________________________________
____________________________________________________________________________

6. Can you explain how a “cut” heals itself? _________________________________________


____________________________________________________________________________
____________________________________________________________________________

Case Scenario 9
Objective: Pre-Analytical Sources of Sample Errors During Collection, Transport, Processing,
and Storage
Case: You find some urine and blood samples that have obviously gone unnoticed for quite a
long while. A check of the sample collection times shows:

● A routine urinalysis sample that is 2 days old


● A blood sample with orders for Infectious Mono testing that is 8 hours old
● A blood sample with orders for a CBC that has been sitting for about 3 weeks.

1. What should be your next step? ________________________________________________


____________________________________________________________________________
____________________________________________________________________________

2. Why? _____________________________________________________________________
____________________________________________________________________________
____________________________________________________________________________

3. Does anyone need to track the cause? Why? _____________________________________


____________________________________________________________________________
____________________________________________________________________________

Case Scenario 10
Objective: Selection of Best Anatomic Site for Blood Draw & Patient Preparation Details
Case: Explain the special considerations that a phlebotomist needs to take into account while
collecting blood from patients in each of these circumstances.
1. What would you do if the patient had a right side mastectomy yesterday? _______________
____________________________________________________________________________
____________________________________________________________________________
NCCT Phlebotomy Review PG109
2. What do you do if the patient develops a hematoma while you are collecting the blood
specimen? ___________________________________________________________________
____________________________________________________________________________
____________________________________________________________________________

3. What would you do if the patient is an outpatient without an identification band? __________
____________________________________________________________________________
____________________________________________________________________________

4. What would you do if the patient has an IV in the left hand? __________________________
____________________________________________________________________________
____________________________________________________________________________

5. What would you do if the patient needs a fingerstick and whose hands are freezing cold? ___
____________________________________________________________________________
____________________________________________________________________________

6. How would you perform a venipuncture on a morbidly obese patient? ___________________


____________________________________________________________________________
____________________________________________________________________________

7. What would you do if the patient is 5 years of age and has orders for a dilatin level to be
collected? ___________________________________________________________________
____________________________________________________________________________
____________________________________________________________________________

8. What would you do if the patient needs a lipid profile drawn? _________________________
____________________________________________________________________________
____________________________________________________________________________

9. What would you do if the patient refuses to let you touch her? ________________________
____________________________________________________________________________
____________________________________________________________________________

10. What would you do if the patient has no good arm/hand veins, but has a good vein in the left
ankle? ______________________________________________________________________
____________________________________________________________________________
____________________________________________________________________________
NCCT Phlebotomy Review PG110
Case Scenario 11
Objective: Risk Factors and Responding to Complications in Phlebotomy
Case: Mr. Simpson is a 45-year-old male patient weighing 200 pounds who has come to the
Outpatient Drawing Station for blood work. As you bring him back to have a blood sample
drawn, he tells you that he “never really minds having blood drawn”, but that sometimes he
tends to “faint” afterward. He really doesn’t look like the type to faint.
1. How would you react to this information? _________________________________________
____________________________________________________________________________
____________________________________________________________________________

2. You ignored Me. Simpson’s warning, drew his blood in a venipuncture chair, and now he has
fainted. What do you do? _______________________________________________________
____________________________________________________________________________
____________________________________________________________________________

3. Mr. Simpson suddenly develops clammy skin, very shallow breathing, and has a very rapid
pulse. Why is this a concern? ___________________________________________________
____________________________________________________________________________
____________________________________________________________________________

Case Scenario 12
Objective: Quality Assurance in Phlebotomy; Accuracy and Reliability in Test Results; and Legal
Implications
Case: You have test orders for blood work on a patient who is coming in later that afternoon.
She needs eight tubes of blood drawn. You relabel the tubes to save time. You put her name,
the date of collection, and your initials on all the tubes and line them up in a test tube rack to
use for the venipuncture when the patient arrives.
1. Is pre-labeling tubes considered good practice? ____________________________________
____________________________________________________________________________
____________________________________________________________________________

2. Why is pre-labeling NOT considered good practice? ________________________________


____________________________________________________________________________
____________________________________________________________________________

3. Who is legally responsible for labeling the patient sample correctly? ____________________
____________________________________________________________________________
____________________________________________________________________________

4. Why is it crucial to have the right patient with the right orders with the right labels for the right
tests? ______________________________________________________________________
____________________________________________________________________________
____________________________________________________________________________
NCCT Phlebotomy Review PG111
Case Scenario 13
Objective: Advanced Infectious Disease Control and Biohazard Handling
Case: A phlebotomist is asked to speak to a class of biology students about your career in
phlebotomy. The teacher wants the following key points addressed in a presentation:

● The risk of health care professionals catching diseases from patients


● The risk of spreading diseases from patient to patient
● The patient’s risk of catching disease from healthcare professionals

1. Explain how to address those key points. _________________________________________


____________________________________________________________________________
____________________________________________________________________________

2. What term is used when a patient acquires an infection while hospitalized that s/he did not
have prior to hospital admission? _________________________________________________
____________________________________________________________________________
____________________________________________________________________________

3. What preventative measure can decrease the chance of acquiring Hepatitis B from
contaminated blood or body fluids? _______________________________________________
____________________________________________________________________________
____________________________________________________________________________
NCCT Phlebotomy Review PG112
Practice Exam - 150Q’s
1. The phlebotomist has a requisition for chemistry and coagulation testing. Which of the
following is the correct tube selection and order of draw?
A. Red, royal blue
B. SST, royal blue
C. Red, light blue
D. Light blue, SST

2. Negligence by a health care professional is considered which of the following?


A. Battery
B. Assault
C. Non-maleficence
D. Malpractice

3. Which of the following is an appropriate alternative to using a rubber tourniquet when


attempting a venipuncture on a known patient who is difficult to obtain a sample from?
A. Elastic bandage
B. Non-adherent gauze
C. Blood pressure cuff
D. Hypoallergenic tape

4. A phlebotomist is using an evacuated tube holder and multi-sample needle to collect a CBC,
PT, and BUN. The correct order of draw for this collection is
A. lavender, light blue, green.
B. green, lavender, light blue.
C. green, light blue, lavender.
D. light blue, green, lavender.

5. The phlebotomist is asked to draw blood from a patient with an IV in the right arm whose left
arm is inaccessible. The phlebotomist may draw without a physician’s approval from the
A. right arm distal to the IV site.
B. right arm proximal to the IV site.
C. foot.
D. IV line.

6. Which of the following information is contraindicated for patient identification at the time of
draw?
A. Insurance number
B. Social security number
C. Date of birth
D. Full name
NCCT Phlebotomy Review PG113
7. Which of the following actions should the phlebotomist take to make a vein more prominent
when attempting to select a venipuncture site?
A. Leave the tourniquet in place and ask the patient to dangle the arm for one minute.
B. Firmly tap the arm several times and palpate with a non-gloved finger.
C. Apply a warm compress to the area for 5 minutes.
D. Apply firm pressure to the arm from the shoulder to the elbow.

8. A phlebotomist puts evacuated tubes filled with blood in plastic transport bags and places
them in the physician office lock box for the reference lab courier who will pick them up in 1-2
hours. The lock box is outside and the temperatures are below freezing. How will freezing
affect the test results?
A. Elevated hemoglobin
B. Elevated hematocrit
C. Decreased red blood cell count
D. Decreased calcium

9. A patient with an order for a blood draw presents with burns covering both arms. Which of
the following is the phlebotomist’s BEST option for collecting the largest amount of blood for this
patient?
A. Microcollection container
B. Filter paper (PKU) card
C. Evacuated tube
D. Capillary tube

10. A CMP is ordered for an older adult patient whose veins continue to collapse despite
drinking water. Which of the following is the BEST collection method for this patient?
A. Evacuated tube system
B. Capillary puncture
C. 23-gauge butterfly needle and syringe
D. 18-gauge needle and syringe

11. While performing a venipuncture in the antecubital space, the patient verbalizes an extreme
amount of pain and the blood in the collection tube is noted to be bright red. Which of the
following sites is MOST likely to have been inadvertently punctured?
A. Brachial artery
B. Ulnar artery
C. Cephalic vein
D. Basilic vein
NCCT Phlebotomy Review PG114
12. A phlebotomist has active symptoms of the common cold. Under which of the following
conditions may he perform venipuncture?
A. The phlebotomist may not perform venipuncture with an active cold.
B. The phlebotomist may perform venipuncture provided he is wearing a mask and afebrile.
C. The phlebotomist may perform venipuncture provided the patient is wearing a mask.
D. The phlebotomist may perform routine venipuncture as usual.

13. Which of the following is the minimum PPE requirement when drawing lab work on a patient
with suspected HIV?
A. Mask only
B. Gloves and mask only
C. Gloves only
D. Gown, gloves, and mask

14. When drawing a blood alcohol specimen, which of the following should be used to clean the
arm?
A. Tincture of iodine
B. ChloraPrep
C. Isopropyl alcohol
D. Benzalkonium chloride

15. The phlebotomist is performing a venipuncture on an apprehensive school age child. Which
of the following approaches should the phlebotomist use to calm the child?
A. Allow child to clean the puncture site and give her a sticker.
B. Tell the child she has to do this, but it won’t hurt.
C. Explain the procedure and reassure the child that the parent can stay with her.
D. Direct the child’s questions to the parent and have the parent explain.

16. A centrifuge functions in which of the following ways?


A. Magnifies cellular blood components
B. Rotates to separate components of a patient's blood
C. Measures the amount of glucose in a patient's blood
D. Slowly warms a refrigerated blood specimen to room temp

17. The rubber sleeve at the end of a ETS venipuncture needle allows for
A. faster draw of the specimen.
B. slower draw of the specimen.
C. drawing a single tube.
D. drawing multiple tubes.
NCCT Phlebotomy Review PG115
18. Which of the following veins would generally be acceptable for routine blood collection by
the phlebotomist?
A. Cephalic
B. Dorsalis pedis
C. Femoral
D. Subclavian

19. When performing a venipuncture, the phlebotomist is stuck with a bloody needle upon
withdrawal from the patient. What is the FIRST thing the phlebotomist should do?
A. Wash the exposed area with soap and water.
B. Seek immediate medical attention.
C. Report to a supervisor.
D. Document the incident.

20. Which of the following statements represents correct patient identification for a phlebotomist
in an outpatient setting?
A. "May I have your name please?"
B. "Is your name Ms. Smith?"
C. "Ms. Smith, I need to see your driver's license."
D. “Is your name Evelyn Smith?”

21. An older adult patient arrives to have blood drawn. While assessing the arms for
venipuncture, the phlebotomist notices scar tissue in the antecubital areas and is unable to
palpate a vein. Which of the following is an appropriate alternative course of action?
A. Use a larger gauge needle to penetrate through the scar tissue.
B. Draw blood from the hand using an evacuated tube system.
C. Perform a capillary puncture on the lateral thumb.
D. Draw blood from the hand using a winged infusion set.

22. How long should pressure be applied on the venipuncture site for a patient on Coumadin
therapy?
A. 3 min
B. 10 min
C. 1 min
D. 5 min

23. The phlebotomist has collected an ammonia level. Which of the following should be done
regarding transport of the specimen?
A. Immerse the specimen in an ice and water slurry.
B. Wrap the specimen in a heel warmer to slow the metabolic process.
C. Maintain the specimen at room temperature.
D. Wrap the specimen in foil to protect from light.
NCCT Phlebotomy Review PG116
24. MOST laboratory testing errors occur during the
A. analytical testing phase.
B. patient prep to lab arrival pre-analytical phase.
C. data processing and record keeping phase.
D. post-analytical phase.

25. The physician has ordered a prothrombin time (PT) and complete blood count (CBC) for an
adult patient with a diagnosis of thrombophlebitis. The phlebotomist should collect the
evacuated tubes in which of the following orders?
A. Lavender, light blue
B. Red, light blue
C. Light blue, red
D. Light blue, lavender

26. Which one of the following specimen types is primarily used to perform CLIA-waived
point-of-care testing for glucose?
A. Plasma
B. Serum
C. Capillary blood
D. Venous blood

27. Gray-topped sodium fluoride tubes should not be used to collect alanine aminotransferase
(ALT) or aspartate aminotransferase (AST) because the anticoagulant
A. destroys many enzymes.
B. prevents hemoconcentration.
C. causes hemolysis.
D. increases lipemia.

28. A patient brings in written lab test results, including a mix of POC and higher complexity
tests, and asks the phlebotomist for help with interpretation. Which of the following should the
phlebotomist do?
A. Explain the interpretation behind the POC tests, but let the patient read the directions with the
higher level tests.
B. Hand the patient pre-printed test explanations downloaded from the CDC web site.
C. Explain the interpretation behind the POC care tests, but refer the patient to the physician for
the higher complexity tests.
D. Refer the patient to the physician for interpretation of all results.
NCCT Phlebotomy Review PG117
29. On the initial attempt the phlebotomist went through the vein, but was able to pull back on
the needle and successfully collect the specimen. Which of the following should the
phlebotomist do NEXT?
A. Explain to the patient that a bruise may be likely, but it should be gone within a few days.
B. Have the patient hold pressure on the site for at least 10 minutes and notify the nurse.
C. Apply a pressure dressing and instruct the patient to leave it on for 24 hours.
D. Apply an ice pack to the site to minimize bleeding and prevent hematoma.

30. If a phlebotomist has questions about disposing of outdated hazardous chemicals, where
should she look to find this information?
A. MSDS
B. NIOSH
C. HIPAA
D. CLIA

31. The phlebotomist is collecting a capillary sample from an infant. Following the collection, the
phlebotomist holds firm pressure on the site until bleeding has stopped. How should the
phlebotomist proceed?
A. Do not bandage.
B. Put a cartoon character bandage on the site.
C. Tape a sterile gauze pad over the site.
D. Bandage with a pressure dressing.

32. Which of the following patient information should a test requisition form include?
A. Sample storage requirements
B. Admission date
C. Latex sensitivity
D. Patient’s blood type

33. Which of the following is a requirement for a healthcare facility to be in compliance with
OSHA?
A. Current certificate of compliance on file
B. Annual employee safety training
C. Payment of annual dues
D. Annual inspections conducted by OSHA

34. The phlebotomist is drawing plain red, purple, green, and blue top tubes. Upon completion
of the blood draw the phlebotomist noticed that he missed an order requiring an SST. The
phlebotomist should share the
A. red
B. purple
C. green
D. blue
NCCT Phlebotomy Review PG118
35. The phlebotomist is asked to draw blood on a non-responsive patient in an emergency
department. This draw would be conducted under
A. implied consent.
B. informed consent.
C. refusal of consent.
D. expressed consent.

36. Small red spots appear near the tourniquet on a patient's arm. This is MOST likely an
indication of
A. petechiae.
B. a collapsed vein.
C. a hematoma.
D. edema.

37. An immunization is currently available for prevention of which of the following bloodborne
diseases?
A. Hepatitis B
B. HIV
C. Hepatitis C
D. Herpes simplex

38. While performing a venipuncture, immediately before removing the needle, the phlebotomist
should
A. gently invert the evacuated tubes.
B. release the tourniquet.
C. apply a gauze pressure dressing.
D. activate the needle safety mechanism.

39. Which of the following tube and additive combinations should be used for a PT/INR test?
A. Light blue, sodium heparin
B. Gray, sodium heparin
C. Gray, sodium citrate
D. Light blue, sodium citrate

40. To help prevent a hematoma at the site after drawing blood, which of the following actions
should the phlebotomist take?
A. Clean the site with an alcohol prep
B. Ask the patient to make a fist
C. Have the patient bend the elbow to hold cotton gauze on the site
D. Apply direct pressure with cotton gauze
NCCT Phlebotomy Review PG119
41. It is appropriate for the phlebotomist to use an alcohol-based hand rub as the only method
of hand hygiene after which of the following activities?
A. After removing soiled gloves
B. After using the restroom
C. Before entering the employee break room
D. Before putting on gloves prior to venipuncture

42. Which of the following actions is an example of compliance with Quality Control (QC)
guidelines for phlebotomy?
A. Ensure reusable supplies in venipuncture trays are recycled.
B. Ensure supplies in venipuncture trays are no more than one month beyond the expiration
date.
C. Check controls on urinalysis reagent strips daily.
D. Check glucometer controls monthly.

43. Which of the following is an example of negative nonverbal communication?


A. Nodding the head
B. Folding the arms across the chest
C. Maintaining eye contact
D. Taking notes

44. Which of the following is the appropriate blood collection device for obtaining a blood spot
collection on an infant?
A. Evacuated tube
B. Lancet
C. Micro-collection container
D. Syringe

45. Which of the following is a site preparation solution required for collecting blood cultures and
blood alcohol levels?
A. Povidone-iodine
B. Hydrogen peroxide
C. Soap and water
D. Isopropyl alcohol

46. If a patient has measles, which of the following is a required additional precaution?
A. Surgical mask
B. Double gloves
C. Fluid resistant apron
D. N95 respirator
NCCT Phlebotomy Review PG120
47. During palpation of a large vessel in the forearm, the phlebotomist detects a vibration
sensation. The phlebotomist should then
A. select a smaller gauge needle for the draw site.
B. use a lower angle of penetration with a butterfly.
C. use a syringe blood collection set up with minimal plunger pressure.
D. choose a different collection site.

48. Instead of obtaining consent from a patient who is mentally competent, the phlebotomist
obtains consent from a family member. This is a violation of
A. the Patient's Bill of Rights.
B. HIPAA.
C. the Stark Law.
D. CLIA Waived Testing.

49. When inspecting a patient’s arm before performing a venipuncture, the most desirable site
appears to be the back of the hand. Which of the following venipuncture methods is MOST
appropriate in this situation?
A. Evacuated tube
B. Needle and syringe
C. Capillary puncture
D. Butterfly needle

50. Which of the following needle gauges is MOST commonly used for venipuncture?
A. 21-23
B. 16-17
C. 18-20
D. 24-25

51. A sharp may be disposed of following a venipuncture once the needle is


A. broken.
B. cut.
C. bent.
D. sheathed.

52. While obtaining blood during a venipuncture, the patient faints. Which of the following
actions should the phlebotomist take initially?
A. Call the patient’s emergency contact on file.
B. Reposition the needle slightly and obtain the ordered specimen.
C. Discontinue the venipuncture and call for help.
D. Apply direct pressure for five full minutes on the venipuncture site.
NCCT Phlebotomy Review PG121
53. The phlebotomist applies the tourniquet, but then has trouble relocating the vein after
several minutes of palpation. Which of the following is an expected outcome in these test
results?
A. Falsely increased platelets from clotting
B. Fibrinolysis from hemodilution
C. Falsely elevated K levels from tissue damage
D. Falsely decreased iron levels from hemolysis

54. After selecting the appropriate location to collect a blood sample, how many inches above
that location should the phlebotomist tie the tourniquet?
A. 5 to 6 inches
B. more than 6 inches
C. 3 to 4 inches
D. 1 to 2 inches

55. Which of the following BEST describes the proper way to clean up a broken glass tube?
A. Place the pieces of glass on a piece of paper and carry the paper to the red biohazard bag.
B. Use a wet towel and wipe over the area so that minute fragments of glass may be picked up.
C. Use a paper towel to pick up the pieces of glass and place them in a trash can.
D. Use a broom and dust pan to sweep up the glass and fragments and place in a sharps
container.

56. Which of the following is the reason for wiping off the FIRST drop of blood from the patient’s
finger with gauze when performing a capillary puncture?
A. The first drop of blood contains too much oxygen.
B. The first drop of blood contains more platelets and clots faster.
C. The first drop of blood may contain traces of tissue fluids.
D. The first drop may be contaminated by the lancet.

57. Which task should a phlebotomist complete prior to daily use of a POC instrument?
A. Perform QC on the instrument.
B. Calibrate the instrument.
C. Run the test patient sample.
D. Ensure that QC was performed within the last month.

58. A phlebotomist has received a requisition to collect specimen for a Bilirubin from a
premature newborn patient. Which of the following is the BEST device for this collection?
A. Winged infusion set
B. Syringe and needle
C. 3.00 mm lancet
D. 1.00 mm lancet
NCCT Phlebotomy Review PG122
59. A patient on coumadin therapy is at the lab for a weekly coagulation test. Which of the
following tubes should the phlebotomist collect for this test?
A. Green top
B. Light blue top
C. Red top
D. Lavender top

60. While performing a capillary stick on a six-month-old infant’s heel, the lancet should be no
longer than
A. 2.0 mm.
B. 3.0 mm.
C. 2.0 cm.
D. 3.0 cm.

61. When inserting a needle during venipuncture, the bevel should be in which of the following
positions in relation to the vein?
A. Right
B. Up
C. Left
D. Down

62. Commitment to privacy, continuity of care, advance directives, and the authority to refuse
treatment are granted according to which of the following?
A. The Patient’s Bill of Rights
B. Good Samaritan Law
C. Americans with Disabilities Act
D. Informed Consent

63. Which of the following standards are being violated when a phlebotomist fails to use proper
handwashing technique?
A. NAACLS
B. OSHA
C. HIPAA
D. CMS

64. A phlebotomist has received a requisition for a blood draw on a patient with a coagulation
disorder. The phlebotomist should be sure to
A. use a blood pressure cuff to apply consistent pressure.
B. hold pressure on the draw site until bleeding has stopped.
C. collect the specimen from the hand to avoid a hematoma.
D. call the doctor to confirm the collection site.
NCCT Phlebotomy Review PG123
65. While performing a venipuncture, the phlebotomist removes the filled tubes, but forgets to
remove the tourniquet prior to needle withdrawal. Which of the following is the MOST likely
result?
A. Hematoma
B. Hemolyzed specimen
C. Hemoconcentrated specimen
D. Syncope

66. The Needlestick Safety and Prevention Act exists to protect healthcare workers from
accidental exposure to
A. hazardous chemicals.
B. biologic toxins.
C. blood borne pathogens.
D. carcinogens.

67. A phlebotomist working in a hospital is collecting blood from a patient with very small,
difficult veins. Three tubes of blood need to be collected. The blue and green tubes fill easily
and completely, but the lavender tube fills incompletely. Changing the lavender tube using the
same draw also results in an incomplete specimen. Which of the following should the
phlebotomist do NEXT?
A. Re-draw the patient using a different site and obtain all three tubes.
B. Re-position the needle and recollect the lavender tube.
C. Submit the samples as drawn.
D. Resubmit the draw order for the next round of draws.

68. Forceful shaking of blood in an evacuated collection tube will cause which of the following
complications?
A. Hemoconcentration
B. Contamination
C. Hemostasis
D. Hemolysis

69. Which of the following specimens require the phlebotomist to follow chain-of-custody
documentation procedures? (Select the two (2) correct answers.)
A. Therapeutic drug monitoring
B. Neonatal screening
C. Drug testing
D. Maternal screening
E. Blood alcohol
NCCT Phlebotomy Review PG124
70. Which of the following actions by the phlebotomist is the FIRST line of defense in preventing
the spread of microorganisms?
A. Perform regular hand hygiene
B. Use aseptic technique when drawing up a medication
C. Wear non-sterile gloves when performing venipuncture
D. Wear sterile gloves when changing a dressing

71. Venipuncture should be avoided on an arm with


A. excessive hair.
B. muscular hypertrophy.
C. alopecia.
D. an elbow splint.
!
72. Which of the following is an example of a pre-analytical error made at the time of collection?
A. Delay in transporting
B. Incomplete requisition
C. Waiting to centrifuge
D. Failing to mix tubes

73. While performing a venipuncture using an evacuated tube, a small amount of blood enters
the tube and then stops. Which of the following conditions is MOST likely to have caused this?
A. Hematoma
B. Sclerosed vein
C. Rolling vein
D. Collapsing vein

74. Which of the following precautions should be used for a patient with pulmonary TB?
A. Contact and airborne
B. Droplet and contact
C. Standard and droplet
D. Airborne and standard

75. The physician orders the following blood work to evaluate a patient for sepsis: CBC, Blood
Culture, and BMP. When using the evacuated tube method for venipuncture, in which of the
following orders should the tubes be drawn?
A. Red top, blood culture, lavender top
B. Blood culture, lavender top, gray top
C. Lavender top, red top, blood culture
D. Blood culture, red top, lavender top
NCCT Phlebotomy Review PG125
76. Which of the following steps is completed FIRST in performing venipuncture?
A. Arrange tubes in order of draw
B. Cleanse site with an alcohol wipe
C. Identify the patient
D. Apply the tourniquet

77. Which of the following organizations makes on-site visits to inspect phlebotomy
laboratories?
A. DEA
B. ASCLS
C. CLIA
D. FDA

78. Which of the following statements by a new phlebotomist indicates the need for further
education on standard precautions?
A. "Sharps containers should be replaced when 2/3 full."
B. "I will perform hand hygiene after removing gloves."
C. "I will wear a gown, gloves, and mask for inpatient collections.“
D. “Antimicrobial wipes may be used for cleaning a collection area at the end of a shift.”

79. The phlebotomist is instructed to perform a capillary stick for newborn screening. Which of
the following collection devices should the phlebotomist use?
A. Butterfly needle and syringe
B. Finger stick lancet
C. Evacuated tube
D. Heel stick lancet

80. Which of the following information is required information on a laboratory test requisition for
a blood specimen?
A. Name of the physician
B. Patient’s social security number
C. Diagnosis
D. Source of the sample

80. Which of the following information is required information on a laboratory test requisition for
a blood specimen?
A. Name of the physician
B. Patient’s social security number
C. Diagnosis
D. Source of the sample
NCCT Phlebotomy Review PG126
82. Which of the following techniques should the phlebotomist use when performing a
venipuncture on a geriatric patient with loose skin?
A. Vigorous massaging of the arm to increase blood flow in the veins
B. A tighter tourniquet to anchor more fragile veins
C. Pulling the skin taut using gentle and sufficient pressure to anchor the vein
D. Using a sharper angle and slower needle insertion to decrease rolling

82. Which of the following techniques should the phlebotomist use when performing a
venipuncture on a geriatric patient with loose skin?
A. Vigorous massaging of the arm to increase blood flow in the veins
B. A tighter tourniquet to anchor more fragile veins
C. Pulling the skin taut using gentle and sufficient pressure to anchor the vein
D. Using a sharper angle and slower needle insertion to decrease rolling

84. A tourniquet was left on the arm for an extensive period of time before the performance of a
venipuncture. Which of the following is MOST likely to occur?
A. The blood specimen will be hemolyzed.
B. The blood specimen will be hemoconcentrated.
C. The patient will have excessive bleeding.
D. The patient will develop a hematoma.

85. The patient on hemodialysis with a left AV shunt has undergone a right sided mastectomy
and has an order for a STAT glucose. After consulting with the physician the phlebotomist is
told to perform a capillary draw. Which of the following sites should the phlebotomist use?
A. Right middle finger
B. Right little finger
C. Left ring finger
D. Left thumb

86. Which of the following is the CLSI recommended micro-collection order of draw for a BMP,
CBC and bilirubin?
A. Green, lavender, red
B. Lavender, green, red
C. Red, green, lavender
D. Lavender, red, green

87. The proper technique to anchor a vein is to use the non-dominant thumb
A. to apply direct downward pressure.
B. and forefinger to stretch the skin above and below the site.
C. to pull the skin toward the wrist.
D. and forefinger to apply direct downward pressure above and below the site.
NCCT Phlebotomy Review PG127
88. When wearing the appropriate personal protective equipment for the particular task or duty,
the phlebotomist is exercising
A. work practice controls.
B. medical asepsis.
C. standard precautions.
D. universal precautions.

89. A phlebotomist receives an order to collect a cold agglutinin specimen. The phlebotomist
should
A. keep the specimen at room temperature.
B. transport the specimen on ice.
C. maintain the specimen at 37 degrees Celsius.
D. protect the specimen from light.

90. A patient notices this symbol on a container in the room and asks what it
means. The phlebotomist should tell the patient the symbol means
A. biohazard material present.
B. emergency eye wash station.
C. isolation precautions required.

91. Which of the following regulations set the minimum standards for medical laboratory practice
and quality?
A. HIPAA
B. DEA
C. OSHA
D. CLIA

92. Which of the following actions is a HIPAA violation?


A. Shredding completed collection lists
B. Putting a note in the laboratory reminding staff that a certain patient has a cold agglutinin
C. Storing a fully stocked collection tray in the clinical laboratory
D. Disposing of extra patient tube labels in a regular trash can
NCCT Phlebotomy Review PG128
93. The phlebotomist notes the patient's arm is scarred in the lateral aspect of the antecubital
fossa, and the other arm is not available. The phlebotomist decides to use a more medial site in
the in the fossa over the basilic vein. Which of the following make this a poor choice? (Select
the two (2) correct answers.)
A. Increased risk of an unsuccessful draw
B. Increased risk of arterial involvement
C. Increased risk of specimen rejection
D. Increased risk of nerve damage
E. Increased risk of hemolysis

94. Capillary blood specimens are MOST likely to be successfully collected from patients that
A. are extremely obese.
B. have peripheral edema.
C. have poor peripheral circulation.
D. are dehydrated.

95. What does the color of an evacuated collection tube represent?


A. The personal protective equipment required for collection
B. The type or absence of an additive
C. A visual identifier for tests expected to be above or below normal range
D. The minimum volume of blood to be obtained

96. Which of the following is the maximum amount of time a phlebotomist can leave a tourniquet
secured in place prior to venous access?
A. Three minutes
B. Two minutes
C. Thirty seconds
D. One minute

97. Which of the following QC measures are within the phlebotomist’s scope of practice?
(Select the three (3) correct answers.)
A. Monitoring temperatures of specimen refrigerators
B. Performing calibration on the chemistry analyzer
C. Daily maintenance on the laboratory hematology instruments
D. Checking the expiration dates of venipuncture supplies
E. Perform daily glucose meter control checks

98. Which of the following POC laboratory tests is within the CLIA defined scope of practice for
a phlebotomist?
A. Cell differential and urine sediment
B. Urine dip and cell differential
C. Strep test and urine reagent strip
D. Strep test and urine sediment
NCCT Phlebotomy Review PG129
99. When performing a capillary puncture on an infant, which of the following is the reason for
using the lateral heel instead of a finger?
A. The small amount of tissue between the skin and bone in the finger makes an injury to the
bone likely.
B. The tissue layers on the finger are more susceptible to permanent nerve damage.
C. The finger produces a slower flow of blood, making it more difficult to obtain a specimen.
D. The blood in the fingertip has been found to contain a higher concentration of hemoglobin
than the heel.

100. Keeping contaminated equipment and supplies away from the phlebotomist’s clothing to
prevent pathogen transmission to the next patient is an example of which of the following?
A. Disinfection
B. Sanitization
C. Medical asepsis
D. Surgical asepsis

101. A phlebotomist must follow transmission based precautions for which of the following
patients?
A. Patient with diabetes
B. Patient with multiple sclerosis
C. Patient with meningitis
D. Patient with lupus

102. A physician has ordered STAT hematology, chemistry, and coagulation tests on a patient.
Which of the following tubes are needed for these procedures? (Select the three (3) correct
answers.)
A. Light blue
B. Yellow
C. Gray
D. Lavender
E. Green

103. Which of the following is included in an exposure control plan?


A. Contaminated needlestick injury evaluation process
B. Dates of each employee's last tetanus booster
C. Maintenance of MSDS and incident report logs
D. Emergency exit markings in compliance with OSHA standards

104. When disposing of hazardous materials, the phlebotomist must adhere to the guidelines
and standards set forth by
A. OSHA.
B. FDA.
C. CMS.
D. CLIA.
NCCT Phlebotomy Review PG130
105. Which of the following is the correct order of PPE removal prior to leaving the exam room
of a patient who is in contact-droplet isolation?
A. Gloves, gown, mask
B. Gown, gloves, mask
C. Gown, mask, gloves
D. Gloves, mask, gown

106. While performing a venipuncture to obtain a CBC and PTT, the phlebotomist punctures the
skin, advances the needle, inserts the tube, but does not see a blood return. The phlebotomist
should NEXT
A. try another tube.
B. adjust the bevel up.
C. advance the needle further.
D. release the tourniquet.

107. Which of the following is outside the scope of practice for a phlebotomist?
A. Recording quality control results on a log sheet.
B. Prioritizing collections based on order requests.
C. Verifying equipment function on point of care testing using a control.
D. Performing CLIA high complexity tests.

108. At which of the following times should a test tube be labeled with the patient’s
identification?
A. After leaving the patient’s room
B. While in the patient’s room, after drawing the blood
C. While in the patient’s room, before drawing the blood
D. Before entering the patient’s room

109. Which of the following would be considered a hospital acquired infection?


A. A patient who contracts a MRSA infection from a venipuncture.
B. An 8-year-old patient admitted with chicken pox.
C. A 72-year-old patient in the ER with flu-like symptoms.
D. A healthcare worker that develops Hepatitis C.

110. A phlebotomist collects a cryofibrinogen specimen, places the specimen in a biohazard


specimen bag, then onto slurry of ice and water and transports it to the laboratory in a timely
manner. In this case the phlebotomist
A. should have kept the specimen at 37˚C.
B. should have protected the specimen from light.
C. should have aliquoted the specimen before transport.
D. performed proper specimen handling.
NCCT Phlebotomy Review PG131
111. Which of the following is the proper way of collecting a blood culture with a winged infusion
set on an adult?
A. Aerobic must be drawn first.
B. A discard tube should be drawn prior to collection.
C. Anaerobic must be drawn first.
D. The maximum blood volume is 3-7ml.

112. For which of the following tubes must the phlebotomist prep the site with povidone iodine or
chlorhexidine?
A. A yellow tube with ACD
B. A light blue tube with sodium citrate
C. A lavender tube with EDTA
D. A yellow tube with SPS

113. The phlebotomist arrives at an inpatient room to collect a blood sample. There is an
isolation cart located outside the room. The phlebotomist washes her hands and puts on the
following personal protective equipment: gown, gloves, and mask. Which of the following types
of precautions are being used for this patient?
A. Contact Droplet
B. Contact Droplet Airborne
C. Contact Airborne
D. Standard Airborne

114. The phlebotomist needs to draw a blood culture, sedimentation rate, PTT, and a glucose
test. Which of the following is the correct order of draw?
A. Gray, light blue, lavender, red
B. Yellow, light blue, lavender, gray
C. Red, yellow, light blue, lavender
D. Yellow, red, lavender, gray

115. A patient has an order for a routine 24-hour urine, with no other special instructions. During
the patient education process, the phlebotomist must tell the patient that the process must begin
A. with an empty bladder and end with a full bladder.
B. with a full bladder and end with an empty bladder.
C. and end with a full bladder.
D. and end with an empty bladder.

116. A phlebotomist must collect type and screen, STAT electrolytes, D-dimer, and estrogen.
Which of the following correctly lists the appropriate additives for these tests in the CLSI
recommended order of draw?
A. Silica, sodium citrate, sodium heparin, potassium EDTA
B. Sodium citrate, silica, sodium heparin, potassium EDTA
C. Sodium citrate, non-additive, lithium heparin, potassium EDTA
D. Non-additive, sodium citrate, lithium heparin, potassium EDTA
NCCT Phlebotomy Review PG132
117. A two-year-old toddler was brought into the lab for Hct and Hgb. Which of the following
sites should be used for the dermal puncture?
A. Index finger of non-dominant hand
B. Middle finger of non-dominant hand
C. Right heel
D. Left great toe

118. Which of the following actions should the phlebotomist take when disposing of a needle
following a venipuncture?
A. Recap the needle and place it immediately in a sharps container.
B. Recap the needle and place it on the blood collection tray.
C. Push the needle guard over the needle until it clicks and then place in a biohazard bag.
D. Place the needle with safety device activated immediately in a sharps container.

119. A 17-month-old presents for a routine capillary stick to determine lead levels. The point of
draw should be the
A. toe.
B. thumb.
C. heel.
D. finger.

120. The phlebotomist has an order to collect a complete blood count with differential, partial
thromboplastin time, basic metabolic panel, and blood cultures. Place the collection tubes in the
correct order of draw. (Click and drag the options in the left column to their correct position in
the right column.)
Lavender
Blood culture bottles
Red
Light blue

121. After the phlebotomist has introduced herself and identified the patient, she should NEXT
A. stabilize the vacuum tube setup.
B. state the purpose for which the physician is ordering the test.
C. reassure the patient that it will not hurt.
D. explain the procedure.

122. For which of the following patients is the use of an adhesive bandage contraindicated?
A. Immunocompromised child
B. Adolescent with sickle cell anemia
C. Newborn with hyperbilirubinemia
D. Adult with hemophilia
NCCT Phlebotomy Review PG133
123. When performing a venipuncture, sudden swelling is observed at the insertion site. Which
of the following actions should the phlebotomist take?
A. Continue obtaining the blood sample. This is a localized reaction to the venipuncture.
B. Release the tourniquet, remove the needle, and immediately apply pressure at the site.
C. Leave the tourniquet in place, immediately remove the needle, and apply pressure at the site.
D. Apply firm pressure above the area of swelling to direct the blood flow into the collection
device.

124. The phlebotomist preps the median cubital vein for a venipuncture procedure. How should
the phlebotomist prep the patient’s skin?
A. Wipe the area lightly from top to bottom in a spiral motion.
B. Wipe the area lightly from bottom to top.
C. Use back-and-forth friction, applied horizontally or vertically.
D. Apply concentric circular friction, working from the outside inward.

125. After performing a venipuncture on a patient the phlebotomist notices the patient is
bending his elbow to hold the gauze in place. This action is contraindicated because bending
the elbow is likely to
A. create a hematoma.
B. cause hemostasis.
C. result in hemorrhage.
D. cause bacterial infection.

126. If a Spanish-speaking patient is not provided a translator, although one was requested,
would be considered a violation of
A. Health Insurance Portability and Accountability Act.
B. patient confidentiality.
C. protected health information.
D. the Patient’s Bill of Rights.

127. A 19-year-old patient offers her arm to the phlebotomist when she comes in with a
venipuncture tray. This is an example of
A. informed consent.
B. parental consent.
C. implied consent.
D. verbal consent.
NCCT Phlebotomy Review PG134
128. In the middle of a blood draw the patient experiences a seizure. Which of the following
should the phlebotomist do after removing the tourniquet and withdrawing the needle? (Select
the two (2) correct answers.)
A. Call for help.
B. Place the patient in Trendelenburg position.
C. Protect the patient’s head and lower her to the floor.
D. Place gauze in the patient’s mouth.
E. Observe the patient in the chair for at least 45 minutes after the seizure.

129. The physician orders a hemoglobin level to be checked on a 2-month-old patient. Which
of the following sites should the phlebotomist use to obtain the specimen?
A. Heel of the foot
B. Antecubital space
C. Lateral fingertip
D. Superficial scalp vein

130. After centrifuging a blood specimen, the phlebotomist notes the serum has a reddish
appearance. Which of the following has occurred?
A. The specimen is hemolyzed
B. A buffy coat has formed and discolored the serum
C. The specimen is contaminated with neutrophils
D. The finding is normal

131. Which of the following actions should the phlebotomist take to decrease the potential for
hemolysis of a blood specimen in an EDTA tube?
A. Allow the tube to stand for 20 minutes prior to processing.
B. Gently invert the tube after collecting the specimen.
C. Store the tube in the refrigerator after the specimen has been collected.
D. Use a 25 gauge needle or smaller to collect the specimen.

132. The phlebotomist applies pressure to the venipuncture site using gauze. After two minutes
he observes that bleeding has not stopped. The NEXT appropriate action would be to
A. elevate the arm while applying pressure.
B. continue to apply pressure while bending the arm at the elbow.
C. notify the physician immediately.
D. wrap the arm with a pressure dressing.

133. The phlebotomist is preparing to remove her gloves after a venipuncture and notices blood
on the gloves. Where should the phlebotomist dispose of her gloves?
A. Biohazard waste container
B. Regular trash container
C. Specimen transport bag
D. Red sharps container
NCCT Phlebotomy Review PG135
134. When a phlebotomist enters a patient’s room to collect a STAT blood sample, a sign above
the bed states that all collections should be from the patient’s central port. Which of the
following should the phlebotomist do NEXT?
A. Exit the patient’s room and proceed directly to the next patient on the collection list.
B. Identify the patient and ask for confirmation that the sign is accurate before proceeding with
the collection.
C. Proceed with sample collection from the central port and alert the nurse when finished.
D. Communicate the test order to the nursing staff and wait for one of them to collect the
sample.

135. While giving a patient an injection, the patient jumped, causing the phlebotomist to get
stuck on the hand with a contaminated needle. After performing thorough hand washing, which
of the following should the phlebotomist do FIRST?
A. Begin infectious disease prophylaxis.
B. Complete an exposure incident report.
C. Report the incident to a supervisor.
D. Obtain patient consent for infectious disease testing.

135. While giving a patient an injection, the patient jumped, causing the phlebotomist to get
stuck on the hand with a contaminated needle. After performing thorough hand washing, which
of the following should the phlebotomist do FIRST?
A. Begin infectious disease prophylaxis.
B. Complete an exposure incident report.
C. Report the incident to a supervisor.
D. Obtain patient consent for infectious disease testing.

137. Using the guidelines for any method of venipuncture, which of the following actions should
the phlebotomist perform FIRST?
A. Position the patient’s arm and cleanse the site with an antiseptic wipe.
B. Apply the tourniquet and thoroughly palpate the selected vein.
C. Review the requirements for collecting and handling the blood specimen.
D. Explain the procedure to the patient.

138. A patient presents to have a diagnostic HIV test drawn. Which of the following forms of
consent must the phlebotomist obtain prior to venipuncture?
A. Implied
B. Informed
C. Expressed
D. Verbal
NCCT Phlebotomy Review PG136
139. Before drawing a patient, the phlebotomist notices a droplet of dried blood on the arm of
the phlebotomy chair. Which of the following should the phlebotomist do?
A. Use a gloved hand to scrape up the blood, clean with disinfectant, then absorb with paper
towel.
B. Moisten the droplet with water, clean with anti-microbial soap, then dry with paper towel.
C. Use an alcohol pad to wipe the dried droplet of blood, then dry with paper towel.
D. Use disinfectant to moisten the droplet, absorb with paper towel, then clean with disinfectant.

140. Which of the following sites should the phlebotomist use for newborn screening blood
collection?
A. Antecubital fossa
B. Lateral or medial plantar surface of the heel
C. Superficial scalp vein
D. Lateral surface of the great toe

141. When performing venipuncture on a patient with a left mastectomy, which of the following
sites should the phlebotomist use?
A. Back of left hand
B. Right antecubital fossa
C. Right saphenous vein
D. Left basilic vein

142. A phlebotomist has orders to collect blood specimens from a patient who has IVs in his
right forearm and in the left antecubital area. Where should the phlebotomist collect a venous
blood specimen?
A. Above the IV in the left arm
B. The middle or ring finger of right hand
C. In the antecubital area of the right arm
D. Distal to the IV in the left arm

143. A phlebotomist has received a requisition to draw a bilirubin on an infant with jaundice.
The phlebotomist should
A. transport the specimen on ice.
B. draw a waste tube.
C. protect the specimen from light.
D. invert the specimen immediately.

143. A phlebotomist has received a requisition to draw a bilirubin on an infant with jaundice.
The phlebotomist should
A. transport the specimen on ice.
B. draw a waste tube.
C. protect the specimen from light.
D. invert the specimen immediately.
NCCT Phlebotomy Review PG137
145. Which of the following is the correct method for transferring blood from a syringe into
evacuated tubes?
A. Uncap the evacuated tubes, place them into a tube rack and transfer blood from the syringe
using a syringe transfer device.
B. Remove the hypodermic needle from the syringe and transfer the blood into capped
evacuated tubes by pushing on the syringe plunger.
C. Uncap the evacuated tubes, place them into a tube rack and transfer the blood from the
syringe through the hypodermic needle.
D. Remove the hypodermic needle from the syringe after activating the safety device and
transfer the blood into capped evacuated tubes using a syringe transfer device.

145. Which of the following is the correct method for transferring blood from a syringe into
evacuated tubes?
A. Uncap the evacuated tubes, place them into a tube rack and transfer blood from the syringe
using a syringe transfer device.
B. Remove the hypodermic needle from the syringe and transfer the blood into capped
evacuated tubes by pushing on the syringe plunger.
C. Uncap the evacuated tubes, place them into a tube rack and transfer the blood from the
syringe through the hypodermic needle.
D. Remove the hypodermic needle from the syringe after activating the safety device and
transfer the blood into capped evacuated tubes using a syringe transfer device.

147. Which of the following is an appropriate draw site for an adult patient with a central line?
A. Dorsal side of the hand
B. Distal phalanx of the fifth finger
C. Palmar side of the wrist
D. Plantar surface of the heel

148. A phlebotomist must collect a specimen for a blood alcohol test on a patient who is allergic
to shell fish. Which of the following antiseptics should the phlebotomist use?
A. Chlorohexidine gluconate
B. Iodine
C. 70% isopropyl alcohol
D. Benzalkonium chloride

149. The phlebotomist is drawing blood on an 88-year-old patient that has multiple skin tears.
Which of the following should be used to secure the 2 x 2 gauze sponge?
A. Band aid
B. Adhesive tape
C. Coban
D. Paper tape
NCCT Phlebotomy Review PG138
150. An inpatient with a left-sided mastectomy has a hematoma in the left arm and an active IV
in the right arm. Which of the following is an appropriate site for venipuncture?
A. Distal to the hematoma
B. Distal to the IV
C. Proximal to the IV
D. Proximal to the hematoma

You might also like